Решите уравнение по теме комбинаторика

Математика — онлайн помощь

Рассмотрим множество, состоящее из n различных элементов. Требуется выбрать из них какие-нибудь k элементов и расположить эти k элементов в каком-либо порядке. Такие упорядоченные последовательности называются размещениями из n элементов по k элементов (упорядоченные – следовательно, последовательности <1,2>и <2,1>— различные размещения).

Если в последовательности нет одинаковых элементов, то говорят о размещении без повторений. Их количество

Если в последовательности допускается наличие одинаковых элементов, то говорят о размещении с повторениями. Их количество

Любое подмножество (неупорядоченное), состоящее из k элементов, называется сочетанием из n элементов по k элементов.

Различные сочетания отличаются друг от друга только самими входящими в них элементами, порядок их следования безразличен, т.е. по условию задачи подмножества <1,2>и <2,1>не различны (соединены).

Число сочетаний без повторений

.

Число сочетаний с повторениями

.

Количество способов переставить элементов в заданном множестве (количество перестановок) вычисляется по формуле

.

При решении простейших комбинаторных задач можно использовать следующую таблицу, определяющую число множеств, состоящих из k элементов, отбираемых из множества, содержащего n элементов

ВыборНеупорядоченныйУпорядоченный
Без повтора
С повтором

Рассмотрим разницу между сочетаниями, размещениями с повторениями, без повторений на следующих примерах.

ПРИМЕРЫ РЕШЕНИЯ ЗАДАЧ

ПРИМЕР 13.2.1 В коробке 6 шаров, пронумерованных от 1 до 6. Из коробки вынимаются друг за другом 3 шара и в этом же порядке записывают полученные цифры. Сколько трехзначных чисел можно таким образом записать?

Решение: По условию задачи подмножества <1;2;3>и <3;1;2>– различные. Повторов в подмножестве быть не может, так как шары не возвращаются в коробку.

.

ПРИМЕР 13.2.2. В коробке 6 шаров пронумерованных от 1 до 6. Из коробки вынимаются 3 шара и записывают число в порядке возрастания цифр. Сколько трехзначных чисел можно таким образом записать?

Решение: По условию задачи подмножества <1;2;3>и <3;2;1>дают число 123, т.е. не являются различными.

.

ПРИМЕР 13.2.3. Условие задачи 2.1 (шары возвращаются в коробку)

Решение: .

ПРИМЕР 13.2.4. Условие задачи 2.2 (шары возвращаются в коробку)

Решение: .

ПРИМЕР 13.2.5. Сколько различных перестановок можно составить из букв слова «комар»?

Решение: .

ПРИМЕР 13.2.6. Сколько различных перестановок можно составить из букв слова «задача»?

Решение: Если бы все шесть букв слова были различны, то число перестановок было бы 6! Но буква «а» встречается в данном слове три раза, и перестановки только этих трех букв «а» не дают новых способов расположения букв. Поэтому число перестановок букв слова «задача» будет не 6!, а в 3! раза меньше, то есть .

ПРИМЕР 13.2.7. В мастерской имеется материал 5 цветов. Поступил заказ на пошив флагов, состоящих из трех горизонтальных полос разного цвета каждый. Сколько таких различных флагов может сшить мастерская?

.

Решение: Флаги отличаются друг от друга как цветом полос, так и их порядком, поэтому разных флагов можно сделать штук.

ПРИМЕР 13.2.8. Сколькими способами можно распределить 5 учеников по 3 параллельным классам?

Решение: Составим вспомогательную таблицу

Номер ученика
Вариант класса

Таким образом, видно, что если для одного ученика существует 3 варианта выбора класса, то для всех 5 учеников существует способов распределения по классам.

ПРИМЕР 13.2.9. На книжной полке помещается 30 томов. Сколькими способами их можно расставить, чтобы при этом первый и второй том не стояли рядом?

Решение: Произведем рассуждения “от обратного”. Тридцать томов на одной полке можно разместить 30! способами.

.

Если 1 и 2 тома должны стоять рядом, то число вариантов расстановки сокращается до , т.к. комбинацию из 1 и 2 тома можно считать за один том, но при этом они могут стоять как (1;2) или (2;1), т.е.

, .

Тогда искомое число способов расстановки есть

ПРИМЕР 13.2.10. Чемпионат, в котором участвуют 16 команд, проводится в два круга, т.е. каждая команда дважды встречается с любой другой. Определить, какое количество встреч следует провести.

Решение: По условию задачи из 16 команд для каждой встречи требуется отобрать 2 команды. В данном случае отбор производится без повтора и порядок отбора не важен, т.е. число вариантов — . Так как команды должны играть дважды число вариантов удваивается, т.е. .

ПРИМЕР 13.2.11. Автомобильная мастерская имеет для окраски 10 основных цветов. Сколькими способами можно окрасить автомобиль, если смешивать от 3 до 7 основных цветов?

Решение: По условию задачи отбор цветов для окраски производится без повтора и порядок отбора не важен, т.е. число вариантов зависит лишь от числа отбираемых для окраски цветов — . Поэтому общее число вариантов есть

.

ПРИМЕР 13.2.12. Турист прошел маршрут из пункта A в пункт B, из B в C и вернулся обратно. Сколько вариантов маршрута существует, если из пункта A в пункт B ведут 3 дороги, а из B в C — 4 и нельзя возвращаться той дорогой, по которой уже прошел?

Решение: Составим схему.

Из рисунка видно, что вариантов маршрута из А в B существует 3, и из B в C – 4, т.е. всего маршрутов .

На обратном пути вариантов маршрута из С в B существует 3 (один уже пройден), и из B в А – 2, т.е. всего возможных обратных маршрутов осталось . Тогда всего вариантов маршрута .

ПРИМЕР 13.2.13. Двенадцати ученикам выданы два варианта контрольной работы. Сколькими способами можно посадить учеников в два ряда по 6 человек, чтобы у сидящих рядом не было одинаковых вариантов, а у сидящих друг за другом был один и тот же вариант?

Решение: Рассуждения произведем несколькими способами

I способ) Первоначально 12 учеников разбивают на 2 группы по 6 человек. Это можно сделать способами.

Затем они могут распределиться по своим рядам согласно схеме

.

Поэтому всего способов распределения учеников будет .

II способ) Первоначально 12 учеников запускают в класс, указывая место, где каждый должен сидеть, например “второй ряд, третье место”. Так как посадочных мест также 12, то всего вариантов распределения 12!
Варианты контрольной работы могут распределиться

“I вариант – I ряд, II вариант – II ряд”

“II вариант – I ряд, I вариант – II ряд”,

Таким образом, всего способов распределения учеников будет .

По приведенным решениям видно, что результаты решений совпадают.

ПРИМЕР 13.2.14. Сколько существует вариантов расположения шести гостей за круглым шестиместным столом?

Решение: Эта задача имеет разные решения и, соответственно разные ответы – в зависимости от того, что понимать под различным расположением гостей за столом. Поэтому исследуем возможные варианты.

Если считать, что нам важно, кто сидит на каком стуле, то это простая задача на перестановки и, следовательно, всего вариантов .

Если же важно не то, кто какой стул занял, а то, кто рядом с кем сидит, то требуется рассмотреть варианты взаимного расположения гостей. В таком случае, расположения гостей, получаемые одно из другого при повороте гостей вокруг стола, фактически являются одинаковыми (смотри рисунок).

Очевидно, что для любого расположения гостей таких одинаковых вариантов, получаемых друг из друга поворотом, — шесть. Тогда общее число вариантов уменьшается в шесть раз и их остается .
В случае же, когда нас интересует только взаимное расположение гостей, то одинаковыми можно считать и такие симметричные расположения, при которых у каждого гостя остаются те же соседи за столом, только левый и правый меняются местами (смотри рисунок).

В такой постановке вопроса общее число различных вариантов расположений гостей уменьшается вдвое и составляет 60.

Отметим, что каждое решение будет считаться правильным при соответствующей постановке задачи.

ПРИМЕР 13.2.15. Семнадцать студентов сдали экзамены по 4 предметам только на “хорошо” и “отлично”. Верно ли утверждение, что хотя бы у двух из них оценки по экзаменационным предметам совпадают?

Решение: Очевидно, что в данном случае речь идет о возможных вариантах вида

Предмет1234
Студент 14455
Студент 25445
Студент 35555
Студент 174454

Данный пример можно решить способом, изложенным в примере 13.1.8., и получить количество вариантов . Приведем другой наглядный способ решения, использующий так называемое “дерево решений”,который представляет все варианты (16 штук) получения экзаменационных оценок.

.

По “дереву решений” видно, что 16 студентов могут сдать экзамены только на “хорошо” и “отлично” так, что их результаты будут отличаться, но если студентов 17, хотя бы одно повторение обязательно будет.

При решении задач комбинаторики используются следующие правила.

Если некоторый объект A может быть выбран из совокупности объектов m способами, а другой объект B может быть выбран nспособами, то:

Правило суммы: выбрать либо A, либо B можно m+n способами.

Правило произведения. Пара объектов (A,B) в указанном порядке может быть выбрана способами.

Примеры и задачи для самостоятельного решения

Решить комбинаторную задачу.

13.2.1.1. В группе 25 студентов. Сколькими способами можно выбрать старосту, заместителя старосты и профорга?

13.2.1.2. В группе 25 студентов. Сколькими способами можно выбрать актив группы, состоящий из старосты, заместителя старосты и профорга?

13.2.1.3. Сколькими способами можно составить список из 10 человек?

13.2.1.4. Сколькими способами из 15 рабочих можно создать бригады по 5 человек в каждой?

13.2.1.5. Буквы азбуки Морзе образуются как последовательности точек и тире. Сколько букв можно составить, используя для кодировки каждой из букв: а) ровно 5 символов? б) не более пяти символов?

13.2.1.6. Кости для игры в домино метятся двумя цифрами. Кости симметричны, и поэтому порядок чисел не существенен. Сколько различных костей можно образовать, используя числа 0,1,2,3,4,5,6?

13.2.1.7. Сколько различных звукосочетаний можно взять на десяти выбранных клавишах рояля, если каждое звукосочетание может содержать от трех до десяти различных звуков?

13.2.1.8. В вазе стоят 10 красных и 5 розовых гвоздик. Сколькими способами можно выбрать из вазы пять гвоздик одного цвета?

13.2.1.9. В некоторых странах номера трамвайных маршрутов обозначаются двумя цветными фонарями. Какое количество различных маршрутов можно обозначить, если использовать фонари восьми цветов?

13.2.1.10. Команда компьютера записывается в виде набора из восьми цифровых знаков – нулей и единиц. Каково максимальное количество различных команд?

13.2.1.11. Десять групп занимаются в десяти расположенных подряд аудиториях. Сколько существует вариантов расписания, при которых группы 1 и 2 находились бы в соседних аудиториях?

13.2.1.12. Два почтальона должны разнести 10 писем по 10 адресам. Сколькими способами они могут распределить работу?

13.2.1.13. Замок открывается только в том случае, если набран определенный трехзначный номер. Попытка состоит в том, что набирают наугад три цифры из заданных пяти. Угадать номер удалось только на последней из всех возможных попыток. Сколько попыток предшествовало удачной?

13.2.1.14. Номер автомобильного прицепа состоит из двух букв и четырех цифр. Сколько различных номеров можно составить, используя 30 букв и 10 цифр?

13.2.1.15. У одного студента есть 7 DVD дисков, а у другого – 9 дисков. Сколькими способами они могут обменять 3 диска одного на 3 диска другого?

13.2.1.16. На вершину горы ведут 7 дорог. Сколькими способами турист может два раза подняться на гору и спуститься с нее, если по одной и той же дороге нельзя проходить дважды?

13.2.1.17. У ювелира было 9 разных драгоценных камней: сапфир, рубин, топаз и т.д. Ювелир планировал изготовить браслет для часов, однако три камня было украдено. Насколько меньше вариантов браслета он может изготовить по сравнению с первоначальными планами?

13.2.1.18. В поезд метро на начальной станции вошли 10 пассажиров. Сколькими способами могут выйти все пассажиры на последующих 6 станциях?

13.2.1.19. За одним столом надо рассадить 5 мальчиков и 5 девочек так, чтобы не было двух рядом сидящих мальчиков и двух рядом сидящих девочек. Сколькими способами это можно сделать?

13.2.1.20. В классе 25 учеников. Верно ли утверждение, что, по крайней мере, у трех из них день рождения в один и тот же месяц?

13.2.1.21. На участке железной дороги расположено 25 станций с билетной кассой в каждой. Касса каждой станции продает билеты до любой другой станции, притом в обоих направлениях. Сколько различных вариантов билетов можно выдать на этом участке?

13.2.1.22. На официальном приеме 50 человек обменялись рукопожатиями. Сколько было сделано рукопожатий?

13.2.1.23. Сколько диагоналей у выпуклого двадцатиугольника?

Уважаемые студенты
На нашем сайте можно получить помощь по всем разделам математики и другим предметам:
✔ Решение задач
✔ Выполнение учебных работ
✔ Помощь на экзаменах

Конспект урока на тему «Решение комбинаторных уравнений» (10 класс)

Сочетаниями без повторений занимался еще великий Паскаль. Он предложил специальную таблицу значений сочетаний без повторений.

Значения представлены в табл. которая называется треугольником Паскаля.

Этот треугольник удивительно красив своей математической красотой, и в его числах можно при желании отыскать различные закономерности. Его можно представить несколько иначе – в виде [26]: равнобедренного треугольника (рис. 10).

Рис. 10. Треугольник Паскаля

Здесь каждое число, кроме единиц на боковых сторонах, является суммой двух чисел, стоящих над ним. Поэтому:

(приводим к общему знаменателю)

(выносим n ! за скобку в знаменателе)

Из этого соотношения и вытекает эффективный способ рекуррентного вычисления значений биномиальных коэффициентов.

Докажем соотношение 1)

Это может использоваться при вычислениях, например, вместо можно вычислить .

Докажем соотношение 2)

Имеется формула, называемая биномом Ньютона, которая использует выражения числа сочетаний с повторениями

где а, b – действительные или комплексные числа.

Коэффициенты называются биномиальными.

Докажем формулу бинома Ньютона по индукции. Доказательство по индукции предполагает:

1) базис индукции – доказательство того, что формула верна для конкретного n , например, для n =1. В нашем случае мы убедились, что формула верна для n =2,3,4. Убедимся, что она верна и для n =1.

2) индукционный шаг. Предполагая, что формула верна для некоторого n , убеждаются, что тогда она верна и для n +1.

3) при истинности шагов 1 и 2 заключают, что формула верна для любого n .

Приступим к индукционному шагу.

Возьмем выражение и получим из него выражение для n +1. Очевидно, что это можно сделать путем умножения на a + b :

Преобразуем полученное выражение:

Для выполнения индукционного шага необходимо показать, что это выражение равно выражению:

Рассмотрим подвыражение выражения (1): и заменим i на i -1.

Получим , т.е. одинаковые коэффициенты перед выражениями , для числа сочетаний в первом и втором подвыражении выражения (1).Это позволит вынести за скобку. Но тогда в не учтен n -й член подвыражения (суммирование идет до n ): тогда, учитывая его, получаем:

Нетрудно видеть, что можно заменить на , кроме того, мы уже доказали, что , поэтому: , что, очевидно, равно выражению:

По индукции получаем, что формула бинома Ньютона верна для любого n .

С использованием бинома Ньютона докажем следствие №1 о количестве подмножеств множества из n элементов:

Рассмотрим следствие №2: .

На использовании бинома Ньютона основано понятие производящей функции – функции, позволяющей получать комбинаторные числа без вычисления факториала:

. Здесь – функция, производящая биномиальные коэффициенты.

При n =1 получаем 1+ x , т.е. (коэффициент перед 1), (коэффициент перед x ).

При n =2 получаем (1+ x ) 2 =1+2 x + x 2 , т.е. и т.д.

Решение комбинаторных уравнений

В комбинаторике тоже могут решаться уравнения, особенностью которых является то, что неизвестная принадлежит множеству натуральных чисел. Например, уравнения вида , xN , где N – множество натуральных чисел или вида:

При решении комбинаторных уравнений часто необходимо уметь выполнять действия с факториалами типа:

Например, в задаче о сравнении пар записей в базе данных из n записей:

, – что и требовалось доказать.

В комбинаторике рассматриваются и другие типовые комбинаторные комбинации, например, разбиения n -элементного множества на k подмножеств, которые называются блоками разбиения. В информатике вычисления на конечных математических структурах часто называют комбинаторными вычислениями, и они требуют комбинаторного анализа для установления свойств и оценки применимости используемых алгоритмов. На рис. 11 приведен один из возможных вариантов классификации основных комбинаций.

Рис. 11. Основные комбинации

Комбинаторные задачи могут быть решены, например, системой компьютерной математики Matematica (3,4) фирмы Wolfram Research , Inc . – пакет расширения «Дискретная математика» ( DiscreteMath ) – комбинаторика и ее функции ( Combinatorica , CombinatorialFunctions ): функции перестановок и сочетаний и др.

Пример 1. Решить уравнение

и представим правую часть в виде

,

откуда следует

x + 3 = 11 и x = 8.

Пример 2. Решить уравнение

Решение. По условию x – целое число, удовлетворяющее неравенством Перепишем уравнение в виде

откуда, после упрощений, получаем

> 4

Пример 3. Решить систему уравнений

Решение. Из второго уравнение находим

Решая последнее уравнение, получаем Но так как не пригодно к решению уравнения, значит x = 18.

Подставляя x = 18 в первое уравнение системы, найдем

18 – y = y + 2, y = 8.

Итак, x = 18, y = 8.

Пример 4. Решить систему уравнений

Решение. Перепишем систему уравнений в виде

или, после упрощений получим

откуда следует x = 2, y = 6.

Решите уравнение (22–25) .

1)=42;

ОДЗ: хN; x > 2

= 42

=-6( исключить – не входит в ОДЗ); =7

=56х;

ОДЗ: хN; x > 3

=

(

((

или -3

1 =0(исключить) или х 2 =-6 (исключить); х 3 =9 (входит в ОДЗ).

3)=30;

ОДЗ: хN; x+1 > 2; х > 1

=

=-6( исключить – не входит в ОДЗ); =5.

4) 5=;

ОДЗ: х

; =

=

=

(20(х-2)-(х+1)(х+2))х

(20х-40-х 2 +2х+х+2)=0 или х=0 или х-1=0

х 2 +3х-20х+42=0 х 1 =0 х 2 =1

х 2 -17х+42=0 корни 0 и 1 не входят в ОДЗ

= 21 ОДЗ: хN; x-3 > 2 ; x > 3

=

— 7х + 12 – 42 = 0

— 7х – 30 = 0

х 1 =10 х 2 = — 3 (не входит в ОДЗ)

2) ; ОДЗ: хN; x > 3

=

=

4х(х-2)(х-1) = 6

х(4х 2 – 12х+8-30х+90)=0

х=0 или 4х 2 – 42х + 98 = 0

2х 2 – 21х + 49 = 0

= 15(х-1) ОДЗ: хN; x > 3

= 15(х-1)

= (х-1)х х 1 = 0 или х 2 = 1 — не входят в ОДЗ

= ОДЗ: хN; x > 4

=

4(х-2)! = 24

х 1 =12; х 2 = — 7(не входит в ОДЗ)

= 43 ОДЗ: хN; x > 5

= 43

х 1 =10; х 2 = 3 (не входит в ОДЗ)

= 89 ОДЗ: хN; x > 7

х 2 – 11х – 60 = 0

х 1 =15; х 2 = — 4(не входит в ОДЗ)

+ = 162 ОДЗ: хN; x > 1

= 162

= 162

2

24х + х 2 + 7х + 12 – 324 = 0

х 2 + 31х – 312 = 0

х 1 =8; х 2 = — 39(не входит в ОДЗ)

=

ОДЗ: x > 4

=

=

(х-2)(х-1)х = 0 или (х-3)-45 = 0

х 1 =2; х 2 = 1 х 3 =0 — не входят в ОДЗ х 4 = 48

= 42 ОДЗ: хN; x > 4

= 12

= 12 х 2 – х – 12 = 0 х 1 =4; х 2 = — 3(не входит в ОДЗ) Ответ: 4.

= 90 ОДЗ:

= 90

х 1 =10; х 2 = — 9(не входит в ОДЗ)

= 132 ОДЗ:

= 132

= 132

x 2 +3 x +2–132 = 0

х 1 =10; х 2 = — 13(не входит в ОДЗ)

= 110 ОДЗ:

= 110

= 110

x 2 +3 x +2– 110 = 0

x 2 +3 x – 108 = 0

х 1 =9; х 2 = — 12(не входит в ОДЗ)

ОДЗ:

решаем методом сложения — 5у = -30; у = 6

ОДЗ: ; у

(х-3)(х-2)(х-1) = 3

4)

Сколько двузначных чисел можно составить из цифр 1. 3, 5, 8, 9 так, чтобы в каждом числе не было одинаковых цифр?

Из 6 открыток надо выбрать 3. Сколькими способами это можно сделать?

Комбинаторика — правила, формулы и примеры с решением

Комбинаторика — это раздел математики, в котором изучаются способы выбора и размещения элементов некоторого конечного множества на основании определенных условий. Выбранные (или выбранные и размещенные) группы элементов называются соединениями. Если все элементы полученного множества разные, получаем соединения без повторений, а если элементы повторяются — соединения с повторениями.

Содержание:

В комбинаторике перестановка — это упорядоченный набор без повторений чисел.

Перестановкой из n элементов называется любое упорядоченное множество из n данных элементов.

Иными словами, это такое множество, для которого указано, какой элемент находится на первом месте, какой — на втором, . какой — на n-м.

Формула числа перестановок

Количество различных шестизначных чисел, которые можно составить из цифр 1, 2, 3, 4, 5, 6, не повторяя эти цифры в одном числе, равно

Размещением из n элементов по k называется любое упорядоченное множество из k элементов, состоящее из элементов данного n-элементного множества.

Формулы для нахождения количества соединений с повторениями обязательны только для классов физико-математического профиля.

Формула числа размещений

Количество различных трехзначных чисел, которые можно составить из цифр 1, 2, 3, 4, 5, 6, если цифры не могут повторяться, равно

Сочетанием без повторений из n элементов по k называется любое k-элементное подмножество данного n-элементного множества.

Формула числа сочетаний

(по определению считают, что

Из 25 учащихся одного класса можно выделить пятерых для дежурства по школе способами, то есть способами.

Некоторые свойства числа сочетаний без повторений

(в частности, )

Схема поиска плана решения простейших комбинаторных задач:

Если элемент А можно выбрать т способами, а элемент В — n способами (при этом выбор элемента А исключает одновременный выбор элемента В), то А или В можно выбрать m + n способами.

Если элемент А можно выбрать m способами, а после этого элемент В — n способами, то А и В можно выбрать способами.

Объяснение и обоснование:

Понятие соединения. Правило суммы и произведения:

При решении многих практических задач приходится выбирать из определенной совокупности объектов элементы, имеющие те или иные свойства, размещать их в определенном порядке и т. д. Поскольку в этих задачах речь идет о тех или иных комбинациях объектов, то такие задачи называют комбинаторными. Раздел математики, в котором рассматриваются методы решения комбинаторных задач, называется комбинаторикой. В комбинаторике рассматривается выбор и размещение элементов некоторого конечного множества на основании определенных условий.

Выбранные (или выбранные и размещенные) группы элементов называют соединениями. Если все элементы полученного множества разные, получаем размещения без повторений, а если элементы могут повторяться — размещения с повторениями. В этом параграфе мы рассмотрим соединения без повторений.

Решение многих комбинаторных задач базируется на двух основных правилах — правиле суммы и правиле произведения.

Правило суммы. Если на тарелке лежат 5 груш и 4 яблока, то выбрать один фрукт (грушу или яблоко) можно 9 способами (5 + 4 = 9). В общем виде справедливо такое утверждение:

  • если элемент А можно выбрать m способами, а элемент В — n способами (при этом выбор элемента А исключает одновременный выбор элемента В), то А или В можно выбрать m + n способами.

Уточним содержание этого правила, используя понятие множеств и операций над ними.

Пусть множество А состоит из m элементов, а множество В -из n элементов. Если множества А и В не пересекаются (то есть ), то множество А В состоит изэлементов.

Правило произведения. Если в киоске продают ручки 5 видов и тетради 4 видов, то выбрать набор из ручки и тетради (то есть пару — ручка и тетрадь) можно 5æ4 = 20 способами (поскольку с каждой из 5 ручек можно взять любую из 4 тетрадей). В общем виде имеет место такое утверждение:

  • если элемент А можно выбрать m способами, а после этого элемент В — n способами, то А и В можно выбрать способами.

Это утверждение означает, что если для каждого из m элементов А можно взять в пару любой из n элементов В, то количество пар равно произведению .

В терминах множеств полученный результат можно сформулировать следующим образом. Если множество А состоит из т элементов, а множество В — из n элементов, то множество всех упорядоченных пар* (а; b), где первый элемент принадлежит множеству А (а ∈ А), а второй  множеству В (b ∈ В), состоит из элементов.

Повторяя приведенные рассуждения несколько раз (или, более строго, используя метод математической индукции), получаем, что правила суммы и произведения можно применять при выборе произвольного конечного количества элементов.

Упорядоченные множества:

При решении комбинаторных задач приходится рассматривать не только множества, в которых элементы можно записывать в любом порядке, но и так называемые упорядоченные множества. Для упорядоченных множеств существенным является порядок следования их элементов, то есть то, какой элемент записан на первом месте, какой на втором и т. д. В частности, если одни и те же элементы записать в разном порядке, то мы получим различные упорядоченные множества. Чтобы различить записи упорядоченного и неупорядоченного множеств, элементы упорядоченного множества часто записывают в круглых скобках, например (1; 2; 3) ≠ (1; 3; 2).

Рассматривая упорядоченные множества, следует учитывать, что одно и то же множество можно упорядочить по-разному. Например, множество из трех чисел <–5; 1; 3>можно упорядочить по возрастанию: (–5; 1; 3), по убыванию: (3; 1; –5), по возрастанию абсолютной величины числа: (1; 3; –5) и т. д.

* Множество всех упорядоченных пар (а; b), где первый элемент принадлежит множеству А (а ∈ А), а второй — множеству В (b ∈ В), называют декартовым произведением множеств А и В и обозначают А × В. Отметим, что декартово произведение В × А также состоит из m*n элементов.

Заметим следующее: для того чтобы задать конечное упорядоченное множество из n элементов, достаточно указать, какой элемент находится на первом месте, какой на втором, . какой на n-м.

Размещения:

Размещением из n элементов по k называется любое упорядоченное множество из k элементов, состоящее из элементов заданного n-элементного множества.

Например, из множества, содержащего три цифры <1; 5; 7>, можно составить следующие размещения из двух элементов без повторений:

(1; 5), (1; 7), (5; 7), (5; 1), (7; 1), (7; 5).

Количество размещений из n элементов по k обозначается (читается: «А из n по k», A — первая буква французского слова arrangement, что означает «размещение, приведение в порядок»). Как видим,

Выясним, сколько всего можно составить размещений из n элементов по k без повторений. Составление размещения представим себе как последовательное заполнение k мест, которые будем изображать в виде клеточек (рис. 21.1). На первое место можем выбрать один из n элементов данного множества (то есть элемент для первой клеточки можно выбрать n способами).

Если элементы нельзя повторять, то на второе место можно выбрать только один элемент из оставшихся, то есть из n – 1 элементов. Теперь уже два элемента использованы и на третье место можно выбрать только один из n – 2 элементов и т. д. На k-е место можно выбрать только один из n – (k –1) = n – k +1 элементов (см. рис. 21.1).

Поскольку требуется выбрать элементы и на первое место, и на второе, . и на k-е, то используем правило произведения и получим следующую формулу числа размещений из n элементов по k:

Например, (что совпадает с соответствующим значением, полученным выше). Аналогично можно обосновать формулу для нахождения числа размещений с повторениями. При решении простейших комбинаторных задач важно правильно выбрать формулу, по которой будут проводиться вычисления. Для этого нужно выяснить следующее:

  1. Учитывается ли порядок следования элементов в соединении?
  2. Все ли заданные элементы входят в полученное соединение?

Если, например, порядок следования элементов учитывается и из n данных элементов в соединении используется только k элементов, то по определению это — размещение из n элементов по k.

После определения вида соединения следует также выяснить, могут ли элементы в соединении повторяться, то есть выяснить, какую формулу необходимо использовать — для количества соединений без повторений или с повторениями.

Примеры решения задач:

Пример:

На соревнования по легкой атлетике приехала команда из 12 спортсменок. Сколькими способами тренер может определить, кто из них побежит в эстафете 4 × 100 м на первом, втором, третьем и четвертом этапах?

Решение:

Количество способов выбрать из 12 спортсменок четырех для участия в эстафете равно количеству размещений из 12 элементов по 4 (без повторений), то есть

Для выбора формулы выясняем ответы на вопросы, приведенные выше. Поскольку для спортсменок важно, в каком порядке они будут бежать, то порядок при выборе элементов учитывается. В полученное соединение входят не все 12 заданных элементов. Следовательно, соответствующее соединение — размещение из 12 элементов по 4 (без повторений, поскольку каждая спортсменка может бежать только на одном этапе эстафеты).

Пример:

Найдите количество трехзначных чисел, которые можно составить из цифр 1, 2, 3, 4, 5, 6, 7, если цифры в числе не повторяются.

Решение:

Количество трехзначных чисел, которые можно составить из семи цифр 1, 2, 3, 4, 5, 6, 7, равно числу размещений из 7 элементов по 3, то есть

Для выбора формулы выясняем, что для чисел, которые мы будем составлять, порядок следования цифр учитывается и не все элементы выбираются (только 3 из заданных семи). Следовательно, соответствующее соединение — размещение из 7 элементов по 3 (без повторений).

Пример:

Найдите количество трехзначных чисел, которые можно составить из цифр 1, 2, 3, 4, 5, 6, 0, если цифры в числе не повторяются.

Выбор формулы проводится таким же образом, как и в задаче 2. Следует учесть, что если число, составленное из трех цифр, начинается цифрой 0, то оно не считается трехзначным. Следовательно, для ответа на вопрос задачи можно сначала из заданных 7 цифр записать все числа, состоящие из 3 цифр (см. задачу 2). Затем из количества полученных чисел вычесть количество чисел, составленных из трех цифр, но начинающихся цифрой 0. В последнем случае мы фактически будем из всех цифр без нуля (их 6) составлять двузначные числа. Тогда их количество равно числу размещений из 6 элементов по 2 (см. решение).

Можно выполнить также непосредственное вычисление, последовательно заполняя три места в трехзначном числе и используя правило произведения. В этом случае для наглядности удобно изображать соответствующие разряды в трехзначном числе в виде клеточек, например так:

Решение:

Количество трехзначных чисел, которые можно составить из семи цифр (среди которых нет цифры 0), если цифры в числе не повторяются, равно числу размещений из 7 элементов по 3, то есть

Но среди данных цифр есть цифра 0, с которой не может начинаться трехзначное число. Поэтому из размещений из 7 элементов по 3 необходимо исключить те размещения, в которых первым элементом является цифра 0. Их количество равно числу размещений из 6 элементов по 2, то есть Следовательно, искомое количество трехзначных чисел равно

Пример:

Решите уравнение

Решение:

ОДЗ: x ∈ N, . Тогда получаем:

На ОДЗ это уравнение равносильно уравнениям:

Тогда x = 0 или x = 5. В ОДЗ входит только x = 5.

Уравнения, в запись которых входят выражения, обозначающие количество соответствующих соединений из x элементов, считаются определенными только при натуральных значениях переменной x. Чтобы выражение имело смысл, следует выбирать натуральные значения (в этом случае также существует и, конечно, Ax 2 ≠ 0). Для преобразования уравнения используем формулы:

Объяснение и обоснование:

Перестановкой из n элементов называется любое упорядоченное множество из n заданных элементов.

Напомним, что упорядоченное множество — это такое множество, для которого указано, какой элемент находится на первом месте, какой на втором, . какой на n-м.

Например, переставляя цифры в числе 236 (в котором множество цифр <2; 3; 6>уже упорядоченное), можно составить такие перестановки без повторений: (2; 3; 6), (2; 6; 3), (3; 2; 6), (3; 6; 2), (6; 2; 3), (6; 3; 2) — всего 6 перестановок* .

Количество перестановок без повторений из n элементов обозначается (P — первая буква французского слова permutation — перестановка). Как видим, = 6.

Фактически перестановки без повторений из n элементов являются размещениями из n элементов по n без повторений, поэтому Произведение обозначается n!. Поэтому полученная формула числа перестановок без повторений из n элементов может быть записана следующим образом:

*Отметим, что каждая из перестановок определяет трехзначное число, составленное из цифр 2, 3, 6 таким образом, что цифры в числе не повторяются.

Например, (что совпадает с соответствующим значением, полученным выше).

С помощью факториалов формулу для числа размещений без повторений

(1)

запишем в другом виде. Для этого умножим и разделим выражение в формуле (1) на произведение тогда

Следовательно, формула числа размещений без повторений из n элементов по k может быть записана так:

(2)

Для того чтобы этой формулой можно было пользоваться при всех значениях k, в частности при k = n – 1 и k = n, договорились считать, что

Например, по формуле (2)

Обратим внимание, что в тех случаях, когда значение n! оказывается очень большим, ответы оставляют записанными с помощью факториалов. Например,

Примеры решения задач:

Для выбора формулы при решении простейших комбинаторных задач достаточно выяснить следующее:

  1. Учитывается ли порядок следования элементов в соединении?
  2. Все ли заданные элементы входят в полученное соединение?

Если, например, порядок следования элементов учитывается и все n заданных элементов используются в соединении, то по определению это перестановки из n элементов.

Пример:

Найдите, сколькими способами можно восемь учащихся построить в колонну по одному.

Решение:

Количество способов равно числу перестановок из 8 элементов, то есть

Для выбора соответствующей формулы выясняем ответы на вопросы, приведенные выше. Поскольку порядок следования элементов учитывается и все 8 заданных элементов выбираются, то искомые соединения — это перестановки из 8 элементов без повторений. Их количество можно вычислить по формуле

Пример:

Найдите количество различных четырехзначных чисел, которые можно составить из цифр 0, 3, 7, 9 (цифры в числе не повторяются).

Решение:

Из четырех цифр 0, 3, 7, 9, не повторяя заданные цифры, можно получить перестановок. Перестановки, начинающиеся с цифры 0, не являются записью четырехзначного числа — их количество . Тогда искомое количество четырехзначных чисел равно

Поскольку порядок следования элементов учитывается и для получения четырехзначного числа надо использовать все элементы, то искомые соединения — это перестановки из 4 элементов. Их количество — . При этом необходимо учесть, что в четырехзначном числе на первом месте не может стоять цифра 0. Таких чисел будет столько, сколько раз мы сможем выполнить перестановки из 3 оставшихся цифр, то есть

Пример:

Имеется десять книг, из которых четыре — учебники. Сколькими способами можно поставить эти книги на полку так, чтобы все учебники стояли рядом?

Решение:

Сначала будем рассматривать учебники как одну книгу. Тогда на полке надо расставить не 10, а 7 книг. Это можно сделать способами. В каждом из полученных наборов книг можно выполнить еще перестановок учебников. По правилу умножения искомое количество способов равно

Задачу можно решать в два этапа. На первом будем условно считать все учебники одной книгой.

Тогда получим 7 книг (6 не учебников + 1 условная книга — учебник). Порядок следования элементов учитывается и используются все элементы (поставить на полку необходимо все книги). Следовательно, соответствующие соединения — это перестановки из 7 элементов. Их количество — .

На втором этапе решения будем переставлять между собой только учебники. Это можно сделать способами. Поскольку нам надо переставить и учебники, и другие книги, то используем правило произведения.

Объяснение и обоснование:

1. Сочетания без повторений:

Сочетанием без повторений из n элементов по k называется любое k-элементное подмножество заданного n-элементного множества.

Например, из множества можно составить следующие сочетания без повторений из трех элементов: , , , .

Количество сочетаний без повторений из n элементов по k элементов обозначается символом (читается: «число сочетаний из п по k» или «це из п по k», С — первая буква французского слова combinaison — сочетание). Как видим,

Выясним, сколько всего можно составить сочетаний без повторений из n элементов по k. Для этого используем известные нам формулы числа размещений и перестановок. Составление размещения без повторений из n элементов по k проведем в два этапа. Сначала выберем k разных элементов из заданного n-элементного множества, не учитывая порядок выбора этих элементов (то есть выберем kэлементное подмножество из n-элементного множества — сочетание без повторений из n-элементов по k). По нашему обозначению это можно сделать способами. После этого полученное множество из k разных элементов упорядочим. Его можно упорядочить способами. Получим размещения без повторений из n элементов по k. Следовательно, количество размещений без повторений из n элементов по k в k! раз больше числа сочетаний без повторений из n элементов по k, то естьОтсюда Учитывая, что по формуле (2) , получаем:

(3)

Например, что совпадает со значением, полученным выше.

Используя формулу (3), можно легко обосновать свойство 1 числа сочетаний без повторений, приведенное в табл. 28.

1) Поскольку то

(4)

Для того чтобы формулу (4) можно было использовать и при k = n, договорились считать, что Тогда

Заметим, что формулу (4) можно получить без вычислений с помощью достаточно простых комбинаторных рассуждений.

Когда мы выбираем k предметов из n, то n – k предметов мы оставляем. Если же, напротив, выбранные предметы оставим, а другие n – k -выберем, то получим способ выбора n – k предметов из n. Мы получили взаимно-однозначное соответствие способов выбора k и n – k предметов из n. Значит, количество одних и других способов одинаково. Но количество одних — , а других , поэтому .

Если в формуле (3) сократить числитель и знаменатель на (n – k)!, то получим формулу, по которой удобно вычислять при малых значениях k:

(5)

Например,

2. Вычисление числа сочетаний без повторений с помощью треугольника Паскаля:

Для вычисления числа сочетаний без повторений можно применять формулу (3): , а можно последовательно вычислять соответствующие значения, пользуясь следующим свойством:

(6)

Для обоснования равенства (6) можно записать сумму, используя формулу (3), и после приведения полученных дробей к общему знаменателю получить формулу для правой части равенства (6) (проделайте это самостоятельно). Также формулу (6) можно получить без вычислений с помощью комбинаторных рассуждений.

— это количество способов выбрать k +1 предмет из n + 1. Подсчитаем это количество, зафиксировав один предмет (назовем его «фиксированным»). Если мы не берем фиксированный предмет, то нам нужно выбрать k +1 предмет из n тех, что остались, а если мы его берем, то нужно выбрать из n тех, что остались, еще k предметов. Первое можно сделать способами, второеспособами. Всего как раз способов, следовательно,

Это равенство позволяет последовательно вычислять значения с помощью специальной таблицы, которая называется треугольником Паскаля. Если считать, что , то он будет иметь вид, представленный в табл. 29.

Каждая строка этой таблицы начинается с единицы и заканчивается единицей

Если какая-либо строка уже заполнена, например третья, то в четвертой строке надо записать на первом месте единицу. На втором месте запишем число, равное сумме двух чисел третьей строки, стоящих над ним левее и правее (поскольку по формуле (6) На третьем месте запишем число, равное сумме двух следующих чисел третьей строки, стоящих над ним левее и правее , и т. д. (а на последнем месте снова запишем единицу).

Примеры решения задач:

Обратим внимание, что, как и раньше, для выбора формулы при решении простейших комбинаторных задач достаточно ответить на вопросы:

  1. Учитывается ли порядок следования элементов в соединении?
  2. Все ли заданные элементы входят в полученное соединение?

Чтобы выяснить, является ли заданное соединение сочетанием, достаточно ответить только на первый вопрос (см. схему в табл. 28). Если порядок следования элементов не учитывается, то по определению это сочетание из n элементов по k элементов.

Пример:

Из 12 членов туристической группы надо выбрать трех дежурных. Сколькими способами можно сделать этот выбор?

Решение:

Количество способов выбрать из 12 туристов трех дежурных равно количеству сочетаний из 12 элементов по 3 (без повторений), то есть

Для выбора соответствующей формулы выясняем ответы на вопросы, приведенные выше. Поскольку порядок следования элементов не учитывается (для дежурных неважно, в каком порядке их выберут), то соответствующее соединение является сочетанием из 12 элементов по 3 (без повторений). Для вычисления можно использовать формулы (3) или (5), в данном случае применяем формулу (3):

Пример:

Из вазы с фруктами, в которой лежат 10 разных яблок и 5 разных груш, требуется выбрать 2 яблока и 3 груши. Сколькими способами можно сделать такой выбор?

Решение:

Выбрать 2 яблока из 10 можно способами. При каждом выборе яблок груши можно выбрать способами. Тогда по правилу произведения выбор требуемых фруктов можно выполнить способами. Получаем

Сначала отдельно выберем 2 яблока из 10 и 3 груши из 5.

Поскольку при выборе яблок или груш порядок следования элементов не учитывается, то соответствующие соединения — сочетания без повторений.

Учитывая, что требуется выбрать 2 яблока и 3 груши, используем правило произведения и перемножим полученные возможности выбора яблок и груш

Бином Ньютона:

Поскольку (при x ≠ 0 и a ≠ 0), то формулу бинома Ньютона можно записать еще и так:

Общий член разложения степени бинома имеет вид

(где ). Коэффициенты называют биномиальными коэффициентaми.

Свойства биномиальных коэффициентов:

  1. Число биномиальных коэффициентов (а следовательно, и число слагаемых) в разложении n-й степени бинома равно n + 1.
  2. Коэффициенты членов, равноудаленных от начала и конца разложения, равны между собой (поскольку )
  3. Сумма всех биномиальных коэффициентов равна
  4. Сумма биномиальных коэффициентов, стоящих на четных местах, равна сумме биномиальных коэффициентов, стоящих на нечетных местах.
  5. Для вычисления биномиальных коэффициентов можно воспользоваться треугольником Паскаля, в котором вычисления коэффициентов основываются на формуле

Объяснение и обоснование:

Бином Ньютона:

Двучлен вида a + x также называют биномом. Из курса алгебры известно, что:

Можно заметить, что коэффициенты разложения степени бинома при n = 1, 2, 3 совпадают с числами в соответствующей строке треугольника Паскаля. Оказывается, что это свойство выполняется для любого натурального n, то есть справедлива формула

(7)

Формулу (7) называют биномом Ньютона. Правая часть этого равенства называется разложением степени бинома, а числа (при k = 0, 1, 2, . n) называют биномиальными коэффициентами.

Общий член разложения степени бинома имеет вид

Обосновать формулу (7) можно, например, с помощью метода математической индукции. (Проведите такое обоснование самостоятельно.)

Приведем также комбинаторные рассуждения для обоснования формулы бинома Ньютона.

По определению степени с натуральным показателем (всего n скобок). Раскрывая скобки, получаем в каждом слагаемом произведение n букв, каждая из которых — а или х. Если, например, в каком-либо слагаемом количество букв x равно k, то количество букв а в нем — n – k, то есть каждое слагаемое имеет вид при некотором k от 0 до n. Покажем, что для каждого такого k число слагаемых anравно , откуда после приведения подобных членов и получаем формулу бинома. Произведение получаем, взяв букву x из k скобок и букву а из n – k тех скобок, которые остались. Разные такие слагаемые получим путем разного выбора первых k скобок, а k скобок из n можно выбрать именно способами. Следовательно, общий член разложения бинома действительно имеет вид где k = 0, 1, 2, . n.

Именно из-за бинома Ньютона числа часто называют биномиальными коэффициентами.

Записывая степень двучлена по формуле бинома Ньютона для небольших значений n, биномиальные коэффициенты можно вычислять с помощью треугольника Паскаля (см. табл. 30).

Например,

Так как , формулу бинома Ньютона можно записать в виде:

(8)

Если в формуле бинома Ньютона (8) заменить x на (–x), то получим формулу возведения в степень разности a – x:

Например, (знаки членов разложения чередуются!).

Свойства биномиальных коэффициентов:

  1. Число биномиальных коэффициентов (а следовательно, и число слагаемых) в разложении n-й степени бинома равно n + 1, поскольку разложение содержит все степени x от 0 до n (и других слагаемых не содержит).
  2. Коэффициенты членов, равноудаленных от начала и конца разложения, равны между собой, поскольку
  3. Сумма всех биномиальных коэффициентов равна

Для обоснования полагаем в равенстве (7) значения a = x = 1 и получаем:

Например,

4. Сумма биномиальных коэффициентов, стоящих на четных местах, равна сумме биномиальных коэффициентов, стоящих на нечетных местах.

Для обоснования возьмем в равенстве (7) значения a = 1, x = –1:

Тогда

Примеры решения задач:

Пример:

По формуле бинома Ньютона найдите разложение степени.

Для нахождения коэффициентов разложения можно использовать треугольник Паскаля (табл. 30) или вычислять их по общей формуле. По треугольнику Паскаля коэффициенты равны: 1, 6, 15, 20, 15, 6, 1. Учитывая, что при возведении разности в степень знаки членов разложения чередуются, получаем:

Для упрощения записи ответа можно избавиться от иррациональности в знаменателях полученных выражений (см. решение) или сначала учесть, что ОДЗ данного выражения: x > 0. Тогда то есть данное выражение можно записать так: и возвести в степень последнее выражение.

Решение:

Пример:

В разложении степени найдите член, содержащий

Решение:

.

Общий член разложения:

По условию член разложения должен содержать , следовательно, Отсюда k = 6.

Тогда член разложения, содержащий , равен

На ОДЗ (b > 0) каждое слагаемое в данном двучлене можно записать как степень с дробным показателем. Это позволит проще записать общий член разложения степени

(где k = 0, 1, 2, . n), выяснить, какой из членов разложения содержит и записать его. Чтобы упростить запись общего члена разложения, запишем:

Всё о комбинаторике

Пусть имеется несколько множеств элементов:

Вопрос: сколькими способами можно составить новое множество взяв из каждого исходного множества по одному элементу? Ответ на этот вопрос дают следующие рассуждения.

Элемент из первого множества можно выбрать способами, элемент из второго – s способами, элемент с можно выбрать способами и т. д. Пару элементов можно составить s способами. Это следует из табл. 1.1, в которой перечислены все способы такого выбора.

Способы выбора трех элементов аbc перечислены в табл. 1.2.

В этой таблице строк и s столбцов. Поэтому искомое число способов выбора трех элементов аbc равно s . Продолжая рассуждать подобным образом, получим следующее утверждение.

Основной комбинаторный принцип. Если некоторый первый выбор можно сделать способами, для каждого первого выбора некоторый второй можно сделать s способами, для каждой пары первых двух – третий выбор можно сделать способами и т.д., то число способов для последовательности таких выборов равно s .

Комбинаторные формулы в прикладных задачах теории вероятностей обычно связывают с выбором элементов («выборкой объема ») из совокупности, состоящей из элементов (элементов «генеральной совокупности»). Различают два способа выбора:

  • а) повторный выбор, при котором выбранный элемент возвращается в генеральную совокупность и может быть выбран вновь;
  • б) бесповторный выбор, при котором выбранный элемент в совокупность не возвращается и выборка не содержит повторяющихся элементов.

При повторном выборе каждый по порядку элемент может быть выбран способами. Согласно комбинаторному принципу, такую выборку можно сделать способами. Например, повторную выборку объема 2 из трех элементов можно сделать 3 2 =9 способами:

В случае бесповторной выборки первый элемент можно выбрать способами, для второго остается возможность выбора, третий элемент можно выбрать способами и т.д. Элемент выборки с номером можно выбрать способом. Согласно комбинаторному принципу, общее число бесповторных выборок объема равно

Число называют числом размещений из элементов по .

Например, существует размещений из трех элементов по два: Отметим, что и в первом случае и во втором выборки отличаются либо составом элементов, либо порядком выбора элементов.

Выделим особо случай, когда один за другим выбраны все элементов. В этом случае выборки имеют один и тот же состав (все элементов) и отличаются только порядком выбора элементов. Поэтому число

называют числом перестановок из элементов.

Например, пять человек могут встать в очередь способами. Три элемента можно переставить способами:

Подсчитаем количество бесповторных выборок объема , которые отличаются друг от друга только составом элементов. Пусть X — число таких выборок. Для каждого набора из элементов можно выбрать порядок их расположения способами. Тогда равно числу способов выбрать различных элементов и выбрать порядок их расположения, т.е. равно числу размещений из элементов по :

Это число называют числом сочетаний из элементов по и обозначают через Если в формуле (1.2) умножить числитель и знаменатель на , то

Например, сочетаний из четырех элементов по два существует . Это

Так как из элементов выбрать элементов можно единственным образом, то откуда следует, что

Величины называют биномиальными коэффициентами. Название связано с формулой бинома Ньютона

Из формулы (1.3) следует, что

Биномиальные коэффициенты образуют так называемый треугольник Паскаля, который имеет вид:

В -й строке треугольника Паскаля располагаются коэффициенты, соответствующие представлению по формуле (1.3). Треугольником удобно пользоваться для нахождения значений . Это значение находится на пересечении -й строки и -го наклонного ряда. Например,

Биномиальные коэффициенты обладают свойством симметрии:

Это наглядно демонстрирует треугольник Паскаля. Равенство (1.4) подтверждает тот очевидный факт, что выбор элементов из n равносилен выбору тех элементов из , которые следует удалить, чтобы остались элементов.

При повторном выборе из элементов число выборок объема , которые отличаются только составом равно Еще раз подчеркнем, что речь идет о выборках, которые отличаются хотя бы одним элементом, а порядок выбора этих элементов во внимание не принимается. Число таких выборок можно подсчитать следующим образом. Между элементами поставим разграничительные знаки, например, нули: Таких знаков (нулей) понадобится . На месте каждого элемента поставим столько единиц, сколько раз предполагается выбрать этот элемент. Например, комбинация означает, что элемент выбран четыре раза, элемент выбран один раз, элемент не выбран, . элемент выбран два раза. Заметим, что в такой записи число единиц равно объему выборки . Для перебора всех возможных комбинаций нужно из мест выбрать место и поставить на них нули, а на остальных местах разместить единицы. Это можно сделать способами.

Совокупность из элементов разделить на групп по элементов соответственно можно способами. Порядок элементов внутри каждой из этих групп не имеет значения.

Пусть – множества, число элементов в каждом из которых равно соответственно Составить множество B из элементов множества А1, элементов множества А2, …, элементов множества Аk, можно, согласно основному комбинаторному принципу, способами.

Для безошибочного выбора комбинаторной формулы достаточно последовательно ответить на вопросы в следующей схеме:

Например, число словарей, необходимых для непосредственного перевода с одного на другой, для пяти языков определяется из следующих рассуждений. Для составления словаря выбираем из пяти языков (= 5) любые два (=2). Выбор бесповторный, причем при выборе важен и состав выбора и порядок выбора. Поэтому искомое число словарей равно

Комбинаторные задачи с решением

Комбинаторика — раздел математики, занимающийся вопросом выбора и расположения элементов некоторого конечного множества в соответствии с заданными условиями.

Рассмотрим примеры задач комбинаторики.

Пример №1

Сколькими способами можно выбрать путь из начала координат 0(0,0) в точку В(6,4), если каждый шаг равен единице, но его можно совершать только вправо или вверх? Сколько таких путей проходит через точку А(2,3)?

Решение. Весь путь занимает 10 шагов (четыре вверх и шесть вправо). Для планирования пути следует решить, какие именно по счету четыре шага следует сделать вверх, а остальные шесть — вправо. Выбор бесповторный и нас интересует только состав выбора. Поэтому в описанных условиях всего путей из точки О в точку В будет

Рассуждая подобным образом легко видеть, что путей из точки О в точку А существует а путь из точки А в точку В можно выбрать способами. По комбинаторному принципу всего путей через точку А существует 10 • 5 = 50.

Пример №2

Сколькими способами можно выбрать путь из начала координат 0(0,0) в точку если каждый шаг равен 1, но его можно совершать только вправо или вверх? Сколько таких путей проходит через точку (См. пример 1.1 и исходные данные.)

Исходные данные к задаче 1.1.

Пример №3

В городе с идеальной прямоугольной планировкой (сеть улиц в этом городе изображена на рис. 1.1) из пункта А выходят человек. Половина из них идет по направлению половина — по направлению Дойдя до первого перекрестка, каждая группа разделяется так, что половина ее идет по направлению половина — по направлению Такое же разделение происходит на каждом перекрестке. Требуется перечислить перекрестки, на которых окажутся люди после прохождения N улиц (отрезков на рис. 1.1), и сколько людей окажется на каждом из этих перекрестков.

Решение. Каждый человек пройдет N улиц и окажется на одном из перекрестков Координаты перекрестков указаны в предположении, что точка А служит началом координат.

На каждом перекрестке для каждого человека производится выбор из двух возможностей: идти в направлении или в направлении Поэтому всего возможных путей будет . Из этого следует, что каждый путь пройдет только один человек.

В пункте окажется столько человек, сколько различных путей ведет в этот пункт из точки А . Чтобы попасть в пункт необходимо из N улиц выбрать бесповторным способом к улиц в направлении . Это можно сделать способами.

Ответ.

Пример №4

Сколькими способами можно одинаковых предметов распределить между лицами так, чтобы каждый получил не менее одного предмета?

Решение. Поставим эти предметы в ряд. Между ними будет промежуток. В любые из этих промежутков поставим разделяющие перегородки. Тогда все предметы разделятся на непустых частей. Первую часть передадим первому лицу, вторую — второму и т.д. Выбрать же промежуток из промежутка можно способами. Заметим, что вообще предметов распределить между лицами можно способами.

Ответ.

Пример 1.4.

Сколькими способами можно распределить 6 яблок, 8 груш и 10 слив между тремя детьми? Сколькими способами это можно сделать так, чтобы каждый ребенок получил по меньшей мере одно яблоко, одну сливу и одну грушу?

Решение. Яблоки в соответствии с формулой (1.5) можно распределить способами, груши — , а сливы способами. По комбинаторному принципу всего способов Если необходимо, чтобы каждый ребенок получил по меньшей мере одно яблоко, одну грушу и одну сливу, то в соответствии с формулой предыдущего примера имеем способов.

Пример №5

Сколько цифр в первой тысяче не содержат в своей записи цифры 5?

Решение. Для записи любой из цифр 000, 001, 002, . 999 необходимо трижды выбрать повторным способом одну из десяти цифр, поэтому и получается всего чисел. Если цифру 5 исключить, то выбор можно производить только из девяти цифр: 0, 1,2, 3, 4, 6, 7, 8, 9. Поэтому всего получится чисел в первой тысяче, в записи которых нет цифры 5.

Пример №6

Сколько шестизначных чисел содержат в записи ровно три различных цифры?

Решение. Заметим, что всего шестизначных чисел имеется , так как первая цифра может быть любой (исключая нуль), а остальные пять могут быть выбраны способами.

Выбрать три ненулевых цифры можно способами. Из выбранных трех цифр можно составить шестизначных чисел, из двух — , а из одной — шестизначное число. По формуле (1.7) получаем, что существует шестизначных чисел, в записи которых есть только три заданные цифры. Поэтому общее число шестизначных чисел, в записи которых имеются три отличные от нуля цифры, равно

Учтем теперь возможность использования нуля. К нулю нужно добавить две цифры, что можно сделать способами. Если, например, были выбраны цифры 0, 2, 5, то первой цифрой должна быть 2 или 5. К этой первой цифре в соответствии с формулой (1.7) можно добавить комбинаций остальных пяти цифр. Тогда всего шестизначных чисел, состоящих из 0, 2, 5 будет Всего же шестизначных чисел, записанных тремя цифрами, среди которых встречается нуль, ровно Всего чисел, удовлетворяющих условиям задачи, имеется

Пример №7

В саду есть цветы десяти наименований (розы, флоксы, ромашки и т. д.).

а) Сколькими способами можно составить букет из пяти цветков (не принимая во внимание совместимость растений и художественные соображения)?

б) Сколькими способами можно составить букет из пяти различных цветков?

в) Сколькими способами можно составить букет из пяти цветков так, чтобы в букете непременно было хотя бы по одному цветку двух определенных наименований

Решение. а) Если запрета на повторение цветков нет, то мы имеем дело с повторным выбором и нас интересует только состав. Поэтому по формуле (1.5) получаем способа.

б) Если цветы должны быть разными, то способ выбора бесповторный и букет можно составить способами.

в) Отберем по одному цветку каждого из двух названных наименований. Три остальных цветка можно выбрать из 10 возможных способами.

Ответ. а) 2002; б) 504; в) 220.

Пример №8

Имеется яблок, груш и персиков. Сколькими способами можно их разложить по двум корзинам? Сколькими способами можно это сделать, если в каждой корзине должно быть хотя бы по одному фрукту всех названных видов (полагаем, что фруктов каждого наименования два или больше)?

Решение. Ясно, что яблоки можно разложить способом (в первую корзину можно не положить яблок совсем, положить одно яблоко, два яблока, …, все яблоки). Те же рассуждения в отношении груш и персиков дают соответственно комбинаций. По комбинаторному принципу всего будет способов.

При ответе на второй вопрос учтем, что следует по одному яблоку сразу положить в каждую из корзин, а остальные яблока раскладывать произвольным образом (в первую корзину либо не добавляем яблок, либо добавляем одно, либо –– два, …, либо – все яблока). Все это можно сделать способами. Те же рассуждения насчет других фруктов и комбинаторный принцип дают следующий результат:

Ответ.

Пример №9

Требуется найти число натуральных делителей натурального числа .

Решение. Разложим на простые множители:

где – различные простые числа. (Например, )

Заметим, что при разделении числа на любые два множителя и простые сомножители распределятся между и . Если сомножитель , в число входит то разложение (1.8) примет вид:

Так что разложение на два сомножителя сводится к разделению каждого из чисел на две части, а это можно сделать способами.

Ответ. .

Пример №10

Сколькими способами легкоатлет, собираясь на тренировку, может выбрать себе пару спортивной обуви, имея 5 пар кроссовок и 2 нары кед?

Очевидно, что выбрать одну из имеющихся пар обуви, кроссовки или кеды, можно 5 + 2 = 7 способами.

Обобщая, приходим к комбинаторному правилу сложения:

  • если некоторый элемент можно выбрать способами, а элемент (независимо от выбора элемента ) — способами, то выбрать илиможно способами.

Это правило справедливо также для трех и более элементов.

Пример №11

В меню школьной столовой предлагается на выбор 4 вида пирожков и 3 вида сока. Сколько разных вариантов выбора завтрака, состоящего из одного пирожка и одного стакана сока, имеется у учащегося этой школы?

Пирожок можно выбрать 4 способами и к каждому пирожку выбрать сок 3 способами (рис. 76). Следовательно, учащийся имеет вариантов выбора завтрака.

Обобщая, приходим к комбинаторному правилу умножения:

  • если некоторый элемент можно выбрать , способами и после каждого такого выбора (независимо от выбора элемента ) другой элемент можно выбрать способами, то пару объектов иможно выбрать способами.

Это правило справедливо также для трех и более элементов.

Пример №12

Сколько трехзначных чисел можно составить из цифр 1, 2, 3, 4, если в числе: 1) цифры не повторяются; 2) цифры могут повторяться?

Решение:

1) Первую цифру можем выбрать 4 способами (рис.77). Так как после выбора первой цифры их останется три (ведь цифры в нашем случае повторяться не могут), то вторую цифру можем выбрать 3 способами.И наконец, третью цифру можем выбрать из оставшихся двух — то есть 2 способами. Следовательно, количество искомых трехзначных у чисел будет равно .

2) Применим комбинаторное правило умножения. Так как цифры в числе могут повторяться, то каждую из цифр искомого числа можно выбрать 4 способами (рис. 78), и тогда таких чисел будет .

Ответ. 1) 24 числа; 2) 64 числа.

Отметим, что решить подобные задачи без применения комбинаторного правила умножения можно только путем перебора всех возможных вариантов чисел, удовлетворяющих условию задачи. Но такой способ решения является слишком долгим и громоздким.

Пример №13

Сколько четных пятизначных чисел можно составить из цифр 5, 6, 7, 8, 9, если цифры в числе не повторяются?

Решение:

Четное пятизначное число можно получить, если последней его цифрой будет 6 или 8. Чисел, у которых последней является цифра 6, будет (рис. 79),

а тех, у которых последней является цифра 8, — также 24. По комбинаторному правилу сложения всего четных чисел будет .

Пример №14

Азбука племени АБАБ содержит всего две буквы — «а» и «б». Сколько слов в языке этого племени состоит: 1) из двух букв; 2) из трех букв?

Решение:

1) аа, ба, аб, бб (всего четыре слова); 2) ааа, ааб, аба, абб, ббб, бба, баб, баа (всего восемь слов).

Заметим, что найденное количество слов соответствует комбинаторному правилу умножения. Так как на каждое место есть два «претендента» — «а» и «б», то слов, состоящих из двух букв, будет , а из трех букв — .

Пример №15

В футбольной команде из 11 игроков надо выбрать капитана и его заместителя. Сколькими способами это можно сделать?

Решение:

Капитаном можно выбрать любого из 11 игроков, а его заместителем — любого из 10 оставшихся игроков. Таким образом (по правилу умножения), имеем разных способов.

Пример №16

В Стране Чудес 10 городов и каждые два из них соединяет авиалиния. Сколько авиалиний в этой стране?

Решение. Так как каждая авиалиния соединяет два города, то одним из них может быть любой из 10 городов, а другим — любой из 9 оставшихся. Следовательно, количество авиалиний равно . Но при этом каждую из авиалиний мы учли дважды. Поэтому всего их будет .

Комбинаторные задачи неразрывно связаны с задачами теории вероятностей, еще одного раздела математики.

В ХIII-ХII в. до н. э. встречаются упоминания о вопросах, близких к комбинаторным. Некоторые комбинаторные задачи решали и в Древней Греции. В частности, Аристоксен из Тарента (IV в. до н. э.), ученик Аристотеля, перечислил различные комбинации длинных и коротких слогов в стихотворных размерах. А Папп Александрийский в IV в. н. э. рассматривал число пар и троек, которые можно получить из трех элементов, допуская их повторения. Некоторые элементы комбинаторики были известны и в Индии во II в. до н. э. Индийцы умели вычислять числа, известные нам как коэффициенты формулы бинома Ньютона. Позднее, в VIII в. н. э., арабы нашли и саму эту формулу, и ее коэффициенты, которые сейчас вычисляют с помощью комбинаторных формул или «треугольника Паскаля».

Свой нынешний вид упомянутые комбинаторные формулы приобрели благодаря средневековому ученому Леви бен Гершону (XIV в.) и французскому математику П. Эригону (XVII в.).

В III в. н. э. сирийский философ Порфирий для классификации понятий составил специальную схему, получившую название «древо Порфирия». Сейчас подобные деревья используются для решения определенных задач комбинаторики в разнообразных областях знаний. Некоторые ранее неизвестные комбинаторные задачи рассмотрел Леонардо Пизанский (Фибоначчи) в своей знаменитой «Книге абака» (1202 г.), в частности, о нахождении наименьшего набора различных гирь, позволяющего взвесить груз с любой целочисленной массой, не превышающей заданного числа. Со времен греческих математиков были известны две последовательности, каждый член которых получали по определенному правилу из предыдущих, — арифметическая и геометрическая прогрессии. А Фибоначчи впервые в одной из задач выразил член последовательности через два предыдущих, используя формулу, которую назвали рекуррентной. В дальнейшем метод рекуррентных формул стал одним из мощнейших для решения комбинаторных задач.

Как ни странно, развитию комбинаторики в значительной степени способствовали азартные игры, которые были очень популярны в XVI в. В частности, вопросами определения разнообразных комбинаций в игре в кости в то время занимались такие известные итальянские математики, как Д. Кардано, H. Тарталья и др. А наиболее полно изучил этот вопрос в XVII в. Галилео Галилей.

Современные комбинаторные задачи высокого уровня сложности связаны с объектами в других отраслях математики: определителями, конечными геометриями, группами, математической логикой и т. п.

Правила суммы и произведения

Вспомните, что в математике любые совокупности называют множествами. Объекты, входящие в множества, называют его элементами. Множества обозначают большими латинскими буквами, а их элементы записывают в фигурных скобках. Считают, что все элементы множества различны.

Например,

Множества бывают конечными и бесконечными. Если множество не содержит ни одного элемента, его называют пустым и обозначают символом

Два множества называют равными, если они состоят из одних и тех же элементов.

Если — часть множества то его называют подмножеством множества и записывают Наглядно это изображают с помощью диаграммы Эйлера (рис. 135, а). В частности, для числовых множеств правильные такие соотношения:

Случается, что множества имеют общие элементы. Если множество содержит все общие элементы множеств и только их, то множество называют пересечением множеств Записывают это так: Диаграммой Эйлера пересечение изображают, как показано на рисунке 135, б. Множество, содержащее каждый элемент каждого из множеств и только эти

элементы, называется объединением множеств Если — объединение множеств то пишут (рис. 135, в).

Разницей множеств называют множество, состоящее из всех элементов множества не принадлежащих множеству Его обозначают Например, если

Говоря «множество», «подмножество», порядок их элементов не учитывают. Говорят, что они не упорядочены. Рассматривают и упорядоченные множества. Так называют множества с фиксированным порядком элементов. Их обозначают не фигурными, а круглыми скобками. Например, из элементов множества можно образовать 6 трёхэлементных упорядоченных множеств:

Как множества, все они равны, как упорядоченные множества — разные.

Существуют задачи, в которых надо определить, сколько различных подмножеств или упорядоченных подмножеств можно образовать из элементов данного множества. Их называют комбинаторными задачами, а раздел математики, в котором рассматривается решение комбинаторных задач, называют комбинаторикой.

Комбинаторика — раздел математики, посвящённый решению задач выбора и расположения элементов некоторого конечного множества в соответствии с заданными правилами.

Рассмотрим два основных правила, с помощью которых решается много комбинаторных задач.

Пример №17

В городе есть два университета — политехнический и экономический. Абитуриенту нравятся три факультета в политехническом университете и два — в экономическом. Сколько возможностей имеет студент для поступления в университет?

Решение:

Обозначим буквой множество факультетов, которые выбрал абитуриент в политехническом университете, а буквой — в экономическом: Поскольку эти множества не имеют общих элементов, то в делом абитуриент имеет возможностей для поступления в университет.

Описанную ситуацию можно обобщить в виде утверждения, которое называется правилом суммы.

Если элемент некоторого множества можно выбрать способами, а элемент множества способами, то элемент из множества или из множества можно выбрать способами.

Правило суммы распространяется и на большее количество множеств.

Пример №18

Планируя летний отдых, семья определилась с местами его проведения: в Одессе — 1, в Евпатории — 3, в Ялте — 2, в Феодосии — 2. Сколько возможностей выбора летнего отдыха имеет семья?

Решение:

Поскольку все базы отдыха разные, то для решения задачи достаточно найти сумму элементов всех множеств, о которых говорится: Следовательно, семья может выбирать отдых из 8 возможных.

Пример №19

От пункта до пункта ведут три тропинки, а от — две. Сколько маршрутов можно проложить от пункта до пункта

Решение:

Чтобы пройти от пункта до пункта надо выбрать одну из трёх тропинок: 1, 2 или 3 (рис. 136). После этого следует выбрать одну из двух других троп: 4 или 5. Всего от пункта до пункта ведут 6 маршрутов, потому что Все эти маршруты можно обозначить с помощью пар:

Обобщим описанную ситуацию.

Если первый компонент пары можно выбрать способами, а . второй — способами, то такую пару можно выбрать способами.

Это — правило произведения, его часто называют основным правилом комбинаторики. Обратите внимание: речь идёт об упорядоченных парах, составленных из различных компонентов.

Правило произведения распространяется и на упорядоченные тройки, четвёрки и любые другие упорядоченные конечные множества. В частности, если первый компонент упорядоченной тройки можно выбрать способами, второй — способами, третий — способами, то такую упорядоченную тройку можно выбрать способами. Например, если столовая на обед приготовила 2 первых блюда — борщ (б) и суп (с ), 3 вторых — котлеты (к), вареники (в), голубцы (г) и 2 десертных — пирожные (п) и мороженое (м), то всего из трёх блюд столовая может предложить 12 различных наборов, поскольку

Описанной ситуации соответствует диаграмма, изображённая на рисунке 137. Такие диаграммы называют деревьями.

Пример №20

Сколько разных поездов можно составить из 6 вагонов, если каждый из вагонов можно поставить на любом месте?

Решение:

Первым можно поставить любой из б вагонов. Имеем 6 выборов. Второй вагон можно выбрать из оставшихся 5 вагонов. Поэтому, согласно правилу умножения, два первых вагона можно выбрать способами. Третий вагон можно выбрать из 4 вагонов, которые остались. Поэтому три первых вагона можно выбрать способами. Продолжая подобные рассуждения, приходим к ответу: всего можно составить различных поездов.

Обратите внимание на решение последней задачи. Оно свелось к вычислению произведения всех натуральных чисел от 1 до 6. В комбинаторике подобные произведения вычисляют часто.

Произведение всех натуральных чисел от 1 до называют факториалом и обозначают

Условились считать, что

Языком теории множеств правила суммы и произведения можно сформулировать следующим образом.

Если пересечение множеств пустое, то количество элементов в их объединении равно сумме количества элементов множеств

Если множества имеют общие элементы, то

Если множества конечны, то количество возможных пар равно произведению количества элементов множеств

Пример №21

В розыгрыше на первенство города по баскетболу принимают участие команды из 12 школ. Сколькими способами могут быть распределены первое и второе места?

Решение:

Первое место может получить одна из 12 команд. После того, как определён обладатель первого места, второе место может получить одна из 11 команд. Следовательно, общее количество способов, которыми можно распределить первое и второе места, равно

Пример №22

Сколько четырёхзначных чисел можно составить из цифр 0,1, 2, 3, 4, 5, если ни одна цифра не повторяется?

Решение:

Первой цифрой числа может быть одна из 5 цифр 1, 2, 3, 4, 5. Если первая цифра выбрана, то вторая может быть выбрана 5-ю способами, третья — 4-мя, четвёртая — 3-мя. Согласно правилу умножения общее число способов равно:

Пример №23

Упростите выражение

Решение:

Размещения и перестановки

Задача:

Сколькими способами собрание из 20 человек может избрать председателя и секретаря?

Решение:

Председателя можно выбрать 20-ю способами, секретаря — из остальных 19 человек — 19-ю способами. По правилу произведения председателя и секретаря собрания могут выбрать способами.

Обобщим задачу. Сколько упорядоченных элементных подмножеств можно составить из различных элементов? На первое место можно поставить любой из данных элементов. На второе место — любой из остальных элементов и т. д. На последнее место можно поставить любой из остальных элементов. Из правила произведения следует, что из данных элементов можно получить -элементных упорядоченных подмножеств.

Например, из 4 элементов упорядоченных двухэлементных подмножеств можно образовать всего

Упорядоченое -элементное подмножество элементного множества называют размещением из элементов Их число обозначают

Из предыдущих рассуждений следует, что и что для любых натуральных

В правой части этого равенства множителей. Поэтому результат можно сформулировать в виде такого утверждения.

Число размещений из элементов по равно произведению последовательных натуральных чисел, наибольшее из которых

Примеры:

Пример №24

Сколькими способами можно составить дневное расписание из пяти разных уроков, если класс изучает 10 различных предметов?

Решение:

Речь идёт об упорядоченных 5-элементных подмножествах некоторого множества, состоящего из 10 элементов.

Это размещения.

Ответ. 30 240 способами.

Число размещений из элементов по можно вычислять и по другой формуле: (проверьте самостоятельно).

Размещение элементов по называют перестановками из элементов. Их число обозначают

Например, из трёх элементов можно образовать 6 различных перестановок: Следовательно,

Подставив в формулу числа размещений получим, что

Число перестановок из элементов равно !

Примеры:

Пример №25

Сколькими способами можно составить список из 10 фамилий?

Решение:

Ответ. 3 628 800 способами.

Некоторые комбинаторные задачи сводятся к решению уравнений, в которых переменная указывает на количество элементов в некотором множестве или подмножестве. Рассмотрим несколько таких уравнений.

Пример №26

Решите уравнение

Решение:

Пользуясь формулой размещений, данное уравнение можно заменить таким:

По условию задачи — натуральное число, поэтому — посторонний корень. Следовательно,

Пример №27

Решите уравнение

Решение:

Запишем выражения через произведения.

Имеем:

Поскольку по смыслу задачи Поэтому последнее уравнение можно сократить на произведение Тогда Но уравнение удовлетворяет только одно значение:

Пример №28

Команда из трёх человек выступает в соревнованиях по художественной гимнастике, в которых принимают участие ещё 27 спортсменок. Сколькими способами могут распределиться места между членами команды, при условии, что на этих соревнованиях ни одно место не делится?

Решение:

Речь идёт об упорядоченных 3-элементных подмножествах множества, состоящего из 30 элементов. Это — размещения.

Пример №29

Сколькими способами можно разместить на полке 5 дисков?

Решение:

Речь идёт об упорядоченных 5-элементных множествах. Искомое количество способов равно

Ответ. 120 способами.

Пример №30

Изображённое на рисунке 140 кольцо раскрашено в 7 цветов. Сколько существует таких колец, раскрашенных теми же цветами только в других последовательностях?

Решение:

Зафиксируем одну какую-нибудь часть кольца, окрашенную одним цветом, б других частей можно раскрасить способами.

Ответ. 720 колец.

Пример №31

Сколько можно составить различных неправильных дробей, числителями и знаменателями которых есть числа 3,5, 7,9,11,13?

Решение:

Способ 1. Дробей, у которых числитель не равен знаменателю, можно составить то есть Из этих дробей только половина — неправильных, то есть — 15.

Неправильными являются также дроби, у которых числитель равен знаменателю. Таких дробей в нашем случае 6. Итак, всего можно составить (дробь).

Способ 2. Если знаменатель неправильной дроби 3, то его числителями могут быть все 6 данных чисел. Если знаменатель 5, то числителями неправильной дроби могут быть 5 чисел (5, 7, 9, 11, 13) и т.д. Наконец, если знаменатель — число 13, то существует только 1 неправильная дробь, со знаменателем 13. Всего таких неправильных дробей существует

Комбинации и бином ньютона

Пусть дано множество из трёх элементов: Его двухэлементных подмножеств (не упорядоченных) существует всего три: Говорят, что существует 3 комбинации из трёх элементов по два. Пишут:

Комбинацией из элементов по называют любое элементное подмножество элементного множества.

Число комбинаций из элементов по обозначают В отличие от размещений, комбинации — подмножества неупорядоченные.

Сравните: При тех же значениях значение меньше Можно также указать, во сколько раз меньше. Каждую элементную комбинацию можно упорядочить способами. В результате из одной комбинации получают размещений (упорядоченных подмножеств) из тех же элементов. Итак,

число элементных комбинаций в раз меньше числа размещений из тех же элементов.

То есть, отсюда

Пример №32

Вычислите:

Решение:

Обратите внимание! Полагают также, что для любого

Пример №33

Сколькими способами из 25 учеников можно выбрать на конференцию двух делегатов?

Решение:

Здесь порядок учеников не имеет значения.

Ответ. 300-ми способами.

Докажем, что для натуральных значений правильно тождество

Доказательство. Пусть дано различных элементов: Всего из них можно образовать различных элементных комбинаций. Это количество комбинаций вычислим другим способом. Из данных элементов, кроме последнего можно образовать комбинаций. Остальные элементные комбинации из всех данных элементов можно образовать, если к каждой комбинации из первых элементов по дописать элемент Таких комбинаций

Следовательно, А это и требовалось доказать.

Такое комбинаторное тождество можно доказать также, воспользовавшись формулой числа комбинаций.

С комбинациями тесно связана формула бинома Ньютона. Вспомните формулу квадрата двучлена:

Умножив получим формулы:

Эти три формулы можно записать и так:

Оказывается, для каждого натурального значения правильна и общая формула:

Это тождество называют формулой бинома Ньютона. а её правую часть разложением бинома Ньютона. Бином — латинское название двучлена. Пользуясь этой формулой, возведём, например, двучлен в пятую степень. Поскольку

Доказать формулу бинома Ньютона можно методом математической индукции.

Доказательство. Предположим, что формула верна для некоторого натурального показателя степени Покажем, что тогда она верна и для следующего за ним значения

Выражения в скобках преобразованы согласно формулы

Следовательно, если формула бинома Ньютона верна для то она правильна и для Для она правильна, так как Поэтому на основе аксиомы математической индукции можно утверждать, что формула верна для любого натурального показателя

Вычислять коэффициенты разложения бинома Ньютона можно не по формуле числа комбинаций, а пользуясь числовым треугольником Паскаля — своеобразным способом вычисления коэффициентов разложения бинома Ньютона

Треугольник Паскаля можно продолжать как угодно далеко. Это следует из тождества Его крайние числа — единицы, а каждое другое равно сумме двух ближайших к нему чисел сверху.

Например, прибавляя числа шестой строки (для получим числа следующей строки (для Следовательно, Общий член разложения бинома можно определить по формуле

  • первый член —
  • второй член —
  • третий член —

Пример №34

В турнире по шашкам приняли участие 5 девушек и 7 юношей. Каждый участник сыграл один раз с каждым другим. Сколько партий было: а) между девушками; б) между юношами; в) между юношами и девушками?

Решение:

а) Речь идёт о 2-элементных подмножествах (неупорядоченных) множества, состоящего из 5 элементов. Это — комбинации.

б) Аналогично

в) Воспользуемся правилом умножения. Поскольку каждой из 5 девушек предстоит сыграть с каждым из 7 юношей, возможных случаев

Пример №35

Для дежурства в столовой приглашают 3-х учеников из 7 класса и 2-х учеников из 10 класса. Сколькими способами это можно сделать, если в 7 классе учится 24 ученика, а в 10 классе — 18.

Решение:

Речь идёт о неупорядоченных подмножествах двух разных множеств. Это — комбинации.

По правилу произведения имеем способов выбрать учащихся для дежурства.

Пример №36

Сколько разных делителей имеет число 1001?

Решение:

Разложим заданное число на простые множители: Если число — делитель числа 1001, то оно должно быть одним из чисел 7, 11,13 (три случая) или любым их произведением. Различных произведений может быть Делителем данного числа есть ещё единица. Следовательно, число 1001 имеет делителей.

Пример №37

Докажите, что выпуклый угольник имеет диагоналей.

Решение:

Отрезков, концами которых являются вершин данного -угольника, существует Среди них есть и сторон данного -угольника. Поэтому диагоналей он имеет

Пример №38

Решение:

Все члены разложения бинома Ньютона такие же, как и члены разложения бинома только их члены с чётными номерами отрицательные.

Пример №39

Найдите номер члена разложения который не содержит

Решение:

Воспользуемся формулой общего члена разложения бинома. Имеем:

По условию задачи то есть Отсюда Следовательно, не содержит шестой член разложения бинома.

Элементы комбинаторики

Решение многих задач теории вероятностей требует знания элементов комбинаторики, основными понятиями которой являются перестановки, размещения и сочетания.

Определение: Перестановки — это комбинации из одних и тех же элементов, отличающиеся только порядком элементов.

Пример:

Даны три числа 1, 2, 3. Определить количество комбинаций из этих элементов, отличающиеся только порядком элементов.

Решение:

Комбинации из данных элементов, отличающиеся только порядком элементов: 123; 132; 213; 231; 321; 312. Всего таких комбинаций Если дано n элементов, то число перестановок O2. Размещения — это комбинации, составленные из n различных элементов по m элементов, которые отличаются либо составом элементов, либо их расположением.

Пример:

Даны три числа 1, 2, 3. Определить количество размещений из этих элементов по два, отличающиеся составом или порядком элементов.

Решение:

Комбинации из данных элементов по два, отличающиеся составом или порядком элементов: 12; 21; 23; 32; 13; 31. Всего таких комбинаций 6. Если дано n элементов, то число размещений по m элементов, которые отличаются либо составом элементов, либо их расположением:

Определение: Сочетания — это комбинации, составленные из n различных элементов по m элементов, которые отличаются друг от друга хотя бы одним элементом.

Пример:

Даны три числа 1, 2, 3. Определить количество размещений из этих элементов по два, отличающиеся хотя бы одним элементом.

Решение:

Комбинации из данных элементов по два, отличающиеся хотя бы одним элементом: 12; 23; 13. Всего таких комбинаций 3. Если дано n элементов, то число сочетаний по m элементов, которые отличаются хотя бы одним элементом:

Пример:

Пусть в урне находится n прономерованных шаров. Определить количество способов, которыми можно извлечь из урны эти шары один за другим.

Решение:

Число способов равно числу различных комбинаций из п элементов, отличающихся только порядком элементов, т.е. числу перестановок:

Пример:

Из колоды, содержащей 36 карт, наугад вынимают 3 карты. Найти вероятность того, что среди выбранных карт окажется один туз.

Решение:

Событие А состоит в том, что среди выбранных карт окажется один туз. Это сложное событие состоит из двух событий: выбирается один туз из четырех, а две другие карты выбираются из оставшихся 32 карт. Следовательно, число случаев, благоприятствующих появлению события A, равно Всего возможных равновероятных исходов, образующих полную группу определяется числом сочетаний из 36 карт по 3 карты, т.е. Таким образом, вероятность события А равна

Арифметика случайных событий

Будем считать, что все события, которые могут произойти в рамках данного эксперимента, располагаются внутри квадрата G, тогда невозможные события располагаются вне квадрата G (Рис. 2):

Рис. 2. Квадрат возможных событий.

Таким образом, достоверное событие определяется внутренней частью квадрата, а невозможное — областью вне квадрата.

Определение: Суммой двух случайных событий А и В называется третье случайное событие С, которое состоит в том, что произойдет (или не произойдет) или событие А, или событие В : С = А + В (Рис. 3).

Определение: Суммой n случайных событий называется случайное событие С, которое реализуется в данном опыте, если произойдет (или не произойдет) или одно событий , или любая их совокупность:

Рис. 3. Сумма случайных событий

Замечание: Если в словесном описании сложного события присутствует разделительный союз “или” между элементарными событиями, то речь идет о сумме этих элементарных событий.

Замечание: Суммой события А и ему противоположного события является достоверное событие т.е. Следовательно, противоположное событие можно записать в виде

Определение: Произведением двух случайных событий А и В называется третье случайное событие С, которое состоит в том, что произойдет (или не произойдет) и событие А, и событие В : (Рис. 4).

Рис. 4. Произведение случайных событий.

Определение: Произведением n случайных событий называется случайное событие С, которое реализуется в данном опыте, если произойдет (или не произойдет) совместная реализация событий

Замечание: Если в словесном описании сложного события присутствует соединительный союз “и” между элементарными событиями, то речь идет о произведении этих элементарных событий.

Пример №40

Пусть имеются передатчик и приемник. Приемник удален от передатчика недостаточно большое расстояние, при котором он может при определенных условиях не принять один из сигналов, переданных передатчиком. Пусть передатчик послал три сигнала. Определить следующие сложные события:

  • а) приемник принят только второй сигнал (событие А );
  • б) приемник принял только один сигнал (событие В);
  • в) приемник принял не менее двух сигналов (2 или 3 сигнала — событие С);
  • г) приемник не принял ни одного сигнала (событие D);
  • д) приемник принял хотя бы один сигнал (событие E).

Решение:

Обозначим через элементарное событие, состоящее в том, что приемник принял сигнал i.

Сложное событие А состоит в том, что приемник не принял первый сигнал и принял второй сигнал, и не принял третий сигнал. Так как между элементарными событиями стоит соединительный союз “и”, то речь идет о их произведении, т.е.

Сложное событие В состоит в том, что приемник принял или первый сигнал, или принял второй сигнал, или принял третий сигнал. Так как между элементарными событиями стоит разделительный союз “или”, то речь идет о сумме сложных событии, т.е.

Рассуждая аналогично, получим выражения для остальных событий: Сложное событие Е содержит в своем словесном описании слова “хотя бы один”, следовательно, оно противоположно событию, содержащему в своем словесном описании слова “ни один”, т.е. событию D:

Теорема сложения вероятностей несовместных событий

Теорема: Если случайные события А и В несовместны, то вероятность их суммы равна сумме вероятностей этих событий, т.е. Р(А + В) = Р(А) + Р(В)

Доказательство: Пусть в данном опыте имеется n равновозможных, элементарных, несовместных событий и пусть в m случаях наступает событие А, а в l случаях-событие В. Тогда появлению события А + В благоприятствует m+l исходов. Поэтому

Следствие: Если имеется N событий, то

Следствие: Если события () образуют полную группу, то

Доказательство: Так как события образуют полную группу равно возможных, элементарных, несовместных событий, то их сумма есть достоверное событие а вероятность достоверного события равна 1.

Следствие: Вероятность суммы противоположных событий равна 1.

Доказательство: В силу того, что события А и ему противоположное событие образуют полную группу несовместных событий, то по следствию вероятность их суммы равна 1.

Замечание: Если сложное событие состоит из суммы элементарных событий, то перед применением теоремы надо определить совместны или несовместны элементарные события.

Пример:

Пусть в урне находится 5 белых шаров, 3 — красных и 4 — зеленых. Из урны наудачу вынули шар. Какова вероятность того, что данный шар цветной?

Решение:

Событие, состоящее в том, что из урны извлечен красный шар, обозначим через А. Событие, состоящее в том, что из урны извлечен зеленый шар, обозначим через В. Тогда извлечение цветного шара есть событие С. Так как события А и В несовместны, т.е. событие С состоит в том, что из урны извлечен или событие А , или событие В, то С = А + В. Используя теорему о сложении вероятностей несовместных событий, получим:

Зависимые и независимые события. Условная и безусловная вероятности

Определение: Случайные события А и В называются независимыми, если появление одного из них не влияет на вероятность появления другого события, в противном случае события называются зависимыми.

Замечание: В этом определении речь идет не о причинно-следственной связи между событиями, а о вероятностной (появление одного из них не влияет на вероятность появления другого события), которая является более общей зависимостью между событиями.

Пример №41

В хранилище находится 10 исправных и 5 неисправных приборов, причем неизвестно, какие из них исправные, а какие — нет. Обозначим событием А — из хранилища взят исправный прибор, а В — взят неисправный прибор. Пусть вначале взят неисправный прибор. Определить вероятности указанных событий с возвращением неисправного прибора на склад и без возвращения неисправного прибора в хранилище.

Решение:

Если неисправный прибор возвращается в хранилище, то события А и В независимы и их вероятности равны Во втором случае, когда неисправный прибор не возвращается на склад, общее количество приборов в хранилище изменилось и стало равным 14, причем неисправных приборов будет храниться 4. Следовательно, произошедшее событие В изменило вероятности события А и В: т.е. при такой организации эксперимента события А и В являются зависимыми.

Определение: Вероятность случайного события называется безусловной, если при ее вычислении на комплекс условий, в которых рассматривается это случайное событие, не накладывается никаких дополнительных ограничений. Безусловная вероятность обозначается

Определение: Вероятность случайного события называется условной, если она вычисляется при условии, что произошло другое случайное событие. Условная вероятность обозначается

Теорема умножения вероятностей

Т.2. Вероятность совместного появления двух случайных событий А и В равна произведению вероятности одного из них на условную вероятность другого события, вычисленную при условии, что первое событие имело место:

Доказательство: Пусть событие А состоит в том, что брошенная точка наугад в квадрат G попадает в область А, которая имеет площадь Событие В состоит в том, что брошенная наугад в квадрат G точка попадает в область В с площадью Пусть весь квадрат имеет площадь S, а область совместного наступления событий имеет площадь (Рис. 5). Тогда вероятность события А равна а события В —

Рис. 5. Совместное наступление зависимых и независимых случайных событий.

Вероятность совместного наступления событий .Условные вероятности того, что произойдут указанные события, определяются по формулам: Таким образом, можно записать, что вероятность совместного наступления событий равна:

Замечание: Если события А и В независимы, то т.е. безусловная и условная вероятности равны между собой.

В связи с вышеприведенным замечанием теорема об умножении вероятностей независимых случайных событий имеет вид:

ТЗ. Вероятность совместного наступления независимых событий равна произведению вероятностей этих событий:

Замечание: Независимость случайных событий всегда взаимная. Если то по теореме откуда следует, что

Следствие: Методом математической индукции теоремы легко обобщается на произведение N зависимых событий:

а теорема — для независимых событий:

Замечание: Если сложное событие представляется в виде произведения элементарных событий, то при вычислении вероятности такого события надо определить, зависимы или независимы эти элементарные события.

Что такое комбинаторика

Понятие множества и его элементов:

  • Элемент а принадлежит множеству А
  • Элемент принадлежит множеству
  • В множестве нет элементов

Множество можно представить как совокупность некоторых объектов, объединенных по определенному признаку. В математике множество — одно из основных неопределяемых понятий. Каждый объект, принадлежащий множеству А, называется элементом этого множества. Множество, не содержащее ни одного элемента, называется пустым множеством и обозначается .

Подмножество

Если каждый элемент множества А является элементом множества В, то говорят, что множество А является подмножеством множества В,

и записывают так: Используется также запись если множество А или является подмножеством множества В, или равно множеству В.

Два множества называются равными, если каждый элемент первого множества является элементом второго множества и, наоборот, каждый элемент второго множества является элементом первого множества.

Пересечение множеств

Пересечением множеств A и В называют их общую часть, то есть множество С всех элементов, принадлежащих как множеству А, так и множеству В

Объединение множеств

Объединением множеств А и В называют множество С, состоящее из всех элементов, принадлежащих хотя бы одному из этих множеств (А или В)

Разность множеств

Разностью множеств А и В называется множество С, которое состоит из всех элементов, принадлежащих множеству А и не принадлежащих множеству В

Если все рассматриваемые множества являются подмножествами некоторого универсального множества U, то разность U \А называется дополнением множества А. Другими словами, дополнением множества А называется множество, состоящее из всех элементов, не принадлежащих множеству А (но принадлежащих универсальному множеству).

Объяснение и обоснование:

Понятие множества

Одним из основных понятий, которые используются в математике, является понятие множества. Для него не дается определения. Можно пояснить, что множеством называют произвольную совокупность объектов, а сами объекты — элементами данного множества. Так, можно говорить о множестве учеников в классе (элементы — ученики), множестве дней недели (элементы — дни недели), множестве натуральных делителей числа 6 (элементы — числа 1, 2, 3, 6) и т. д.

В курсах алгебры и алгебры и начал анализа чаще всего рассматривают множества, элементами которых являются числа, и поэтому их называют числовыми множествами.

Как правило, множества обозначают прописными буквами латинского алфавита. Например, если множество М состоит из чисел 1; 2; 3, то его обозначают так: М = <1; 2; 3>. Тот факт, что число 2 входит в это множество (является элементом данного множества М) записывается с помощью специального значка следующим образом: ; а то, что число 5 не входит в это множество (не является элементом данного множества), записывается так:

Можно рассматривать также множество, не содержащее ни одного элемента, — пустое множество.

Например: множество простых делителей числа 1 — пустое множество.

Для некоторых множеств существуют специальные обозначения. Так, пустое множество обозначается символом, множество всех натуральных чисел — буквой N, множество всех целых чисел — буквой Z, множество всех рациональных чисел — буквой Q, а множество всех действительных чисел — буквой R.

Множества бывают конечными и бесконечными в зависимости от того, какое количество элементов они содержат. Так, множества А = <7>и М = <1; 2; 3>— конечные потому, что содержат конечное число элементов, а множества N, Z, Q, R — бесконечные.

Множества задают или с помощью перечисления их элементов (это можно сделать только для конечных множеств), или с помощью описания, когда задается правило (характеристическое свойство), которое позволяет определить, принадлежит или нет данный объект рассматриваемому множеству. Например, А = <-1; 0; 1>(множество задано перечислением элементов), В — множество четных целых чисел (множество задано характеристическим свойством элементов множества). Последнее множество иногда записывают так: — четное целое число> или так: — здесь после вертикальной черточки записано характеристическое свойство.

В общем виде запись множества с помощью характеристического свойства можно обозначить так: — характеристическое свойство. Например,

Равенство множеств

Пусть А — множество цифр трехзначного числа 312, то есть А = <3; 1; 2>, а В — множество натуральных чисел, меньших четырех, то есть В = <1; 2; 3>. Поскольку эти множества состоят из одних и тех же элементов, то они считаются равными. Это записывают так: А = В.

Для бесконечных множеств таким способом (сравнивая все элементы) установить их равенство невозможно. Поэтому в общем случае равенство множеств определяется следующим образом.

Два множества называются равными, если каждый элемент первого множества является элементом второго множества и, наоборот, каждый элемент второго множества является элементом первого множества.

Из приведенного определения равенства множеств следует, что в множестве одинаковые элементы не различаются. Действительно, например, <1; 2; 2>= <1; 2>, поскольку каждый элемент первого множества (1 или 2) является элементом второго множества и, наоборот, каждый элемент второго множества (1 или 2) является элементом первого. Поэтому, записывая множество, чаще всего каждый его элемент записывают только один раз.

Подмножество

Если каждый элемент множества А является элементом множества В, то говорят, что множество А является подмножеством множества В.

Это записывают следующим образом:

Например, (поскольку любое натуральное число — целое), (поскольку любое целое число — рациональное), (поскольку любое рациональное число — действительное).

Полагают, что всегда, то есть пустое множество является подмножеством любого множества.

Иногда вместо записи используется также запись , если множество А является подмножеством множества В или равно множеству В. Например, можно записать, что .

Сопоставим определение равенства множеств с определением подмножества. Если множества А и В равны, то: 1) каждый элемент множества А является элементом множества В, следовательно, А — подмножество В; 2) каждый элемент множества В является элементом множества А, следовательно, В — подмножество Таким образом,

два множества равны, если каждое из них является подмножеством другого.

А = В означает то же, что

Иногда соотношения между множествами удобно иллюстрировать с помощью кругов (которые часто называют кругами Эйлера-Венна). Например, рисунок 118 иллюстрирует определение подмножества, а рисунок 119-отношения между множествами

Операции над множествами

Над множествами можно выполнять определенные действия: находить их пересечение, объединение, разность. Дадим определение этих операций и проиллюстрируем их с помощью кругов.

Пересечением множеств А и В называют их общую часть, то есть множество С всех элементов, принадлежащих как множеству А, так и множеству В.

Пересечение множеств обозначают знаком (на рисунке 120 приведена иллюстрация и символическая запись определения пересечения множеств).

Например, если А = <2; 3; 4>, В = <0; 2; 4; 6>, то

Объединением множеств А и В называют множество С, состоящее из всех элементов, принадлежащих хотя бы одному из этих множеств (А или В).

Объединение множеств обозначают знаком U (на рисунке 121 приведена иллюстрация и символическая запись определения объединения множеств).

Например, для множеств А и В из предыдущего примера Если обозначить множество иррациональных чисел через М, то М U Q = R. Разностью множеств А и В называется множество С, состоящее из всех элементов, которые принадлежат множеству А и не принадлежат множеству В.

Разность множеств обозначают знаком \. На рисунке 122 приведена иллюстрация и символическая запись определения разности множеств.

Например, если А = <1; 2; 3>, В = <2; 3; 4; 5>, то А\В = <1>, а В \ А = <4; 5>. Если В — подмножество А, то разность А \ В называют дополнением множества В до множества А (рис. 123).

Например, если обозначить множество иррациональных чисел через М, то R \ Q = М: множество М иррациональных чисел дополняет множество Q рациональных чисел до множества R всех действительных чисел.

Все множества, которые мы рассматриваем, являются подмножествами некоторого так называемого универсального множества U. Его обычно изображают в виде прямоугольника, а все остальные множества — в виде кругов внутри этого прямоугольника (рис. 124). Разность U \А называется дополнением множества А.

Дополнением множества А называется множество, состоящее из всехэлементов, не принадлежащих множеству А (но принадлежащих универсальному множеству U).

Дополнение множества А обозначается (можно читать: «А с чертой»). Например, если U = R и А = [0; 1], то Для этого примера удобно использовать традиционную иллюстрацию множества действительных чисел на числовой прямой (рис. 125).

Комбинаторика и Бином Ньютона

Элементы комбинаторики:

Комбинаторика — раздел математики, в котором изучаются способы выбора и размещения элементов некоторого конечного множества на основании некоторых условий. Выбранные (или выбранные и размещенные) группы элементов называются Соединения с повторениямими.

Если все элементы полученного множества разные — получаем соединения без повторений, а если в полученном множестве элементы повторяются, то получаем соединения с повторениями*.

Перестановкой из п элементов называется любое упорядоченное множество из элементов.

Иными словами, это такое множество, для которого указано, какой элемент находится на первом месте, какой — на втором. какой — на п-м.

*Формулы для нахождения количества соединений с повторениями являются обязательными только для классов физико-математического профиля. Формула числа перестановок (читается: «Эн факториал»)

Количество различных шестизначных чисел, которые можно составить из цифр 1, 2, 3, 4, 5, 6, не повторяя эти цифры в одном числе, равно

Размещением из элементов по называется любое упорядоченное множество из элементов, состоящее из элементов -элементного множества Формула числа размещений

Количество различных трехзначных чисел, которые можно составить из цифр 1,2,3, 4, 5, 6, если цифры не могут повторяться, равно

Сочетанием без повторений из элементов по называется любое -элементное подмножество -элементного множества Формула числа сочетаний

(по определению считают, что )

Из класса, состоящего из 25 учащихся, можно выделить 5 учащихся для дежурства по школе способами, то есть способами. Некоторые свойства числа сочетаний без повторений

Схема решения комбинаторных задач

Если элемент А можно выбрать способами, а элемент В — способами, то А или В можно выбрать способами.

Если элемент А можно выбрать способами, а после этого элемент В — способами, то А и В можно выбрать способами. Выбор формулы

Учитывается ли порядок следования элементов в соединении?

Все ли элементы входят в соединение?

без повторений с повторениями без повторений с повторениями без повторений с повторениями

Объяснение и обоснование:

Понятие соединения

При решении многих практических задач приходится выбирать из определенной совокупности объектов элементы, имеющие те или иные свойства, размещать эти элементы в определенном порядке и т. д. Поскольку в этих задачах речь идет о тех или иных комбинациях объектов, то такие задачи называют комбинаторными. Раздел математики, в котором рассматриваются методы решения комбинаторных задач, называется комбинаторикой. В комбинаторике рассматривается выбор и размещение элементов некоторого конечного множества на основании определенных условий.

Выбранные (или выбранные и размещенные) группы элементов называют соединениями. Если все элементы полученного множества разные — получаем размещения без повторений, а если в полученном множестве элементы могут повторяться, то получаем размещения с повторениями. Рассматриваются соединения без повторений, а соединения с повторениями.

Решение многих комбинаторных задач базируется на двух основных правилах — правиле суммы и правиле произведения.

Правило суммы

Если на тарелке лежит 5 груш и 4 яблока, то выбрать один фрукт (то есть грушу или яблоко) можно 9 способами (5 + 4 = 9). В общем виде имеет место такое утверждение:

  • если элемент А можно выбрать способами, а элемент В — способами, то А или В можно выбрать способами.

Правило произведения

Если в киоске продают ручки 5 видов и тетради 4 видов, то выбрать набор из ручки и тетради (то есть пару — ручка и тетрадь) можно 5 • 4 = 20 способами (поскольку с каждой из 5 ручек можно взять любую из 4 тетрадей). В общем виде имеет место такое утверждение:

  • если элемент А можно выбрать m способами, а после этого элемент В — способами, то А и В можно выбрать m • п способами.

Это утверждение означает, что если для каждого из т элементов А можно взять в пару любой из элементов В, то количество пар равно произведению

Повторяя приведенные рассуждения несколько раз (или, иначе говоря, используя метод математической индукции), получаем, что правила суммы и произведения можно применять при выборе произвольного конечного количества элементов.

Следовательно, если приходится выбирать или первый элемент, или второй, или третий и т. д. элемент, количества способов выбора каждого еле-мента складывают, а когда приходится выбирать набор, в который входят и первый, и второй, и третий, и т. д. элементы, количества способов выбора каждого элемента перемножают.

Упорядоченные множества

При решении комбинаторных задач приходится рассматривать не только множества, в которых элементы можно записывать в любом порядке, но и так называемые упорядоченные множества. Для упорядоченных множеств существенным является порядок следования их элементов, то есть то, какой элемент записан на первом месте, какой на втором и т. д. В частности, если одни и те же элементы записать в разном порядке, то мы получим различные упорядоченные множества. Чтобы различить записи упорядоченного и неупорядоченного множеств, элементы упорядоченного множества часто записывают в круглых скобках, например

Рассматривая упорядоченные множества, следует учитывать, что упорядоченность не является свойством самого неупорядоченного множества (из которого мы получили упорядоченное), поскольку одно и то же множество можно по-разному упорядочить. Например, множество из трех чисел <-5; 1; 3>можно упорядочить по возрастанию: (-5; 1; 3), по убыванию: (3; 1; — 5), по возрастанию абсолютной величины числа: (1; 3; -5) и т. д.

Будем понимать, что для того чтобы задать конечное упорядоченное множество из п элементов, достаточно указать, какой элемент находится на первом месте, какой на втором, . какой на п-м.

Размещения

Размещением из элементов по называется любое упорядоченное множество из элементов, состоящее из элементов -элементного множества.

Например, из множества, содержащего три цифры <1; 5; 7>, можно составить следующие размещения из двух элементов без повторений: (1;5),(1;7),(5; 7), (5; 1), (7; 1), (7; 5).

Количество размещений из элементов по обозначается (читается: «А из по », А — первая буква французского слова arrangement, что означает «размещение, приведение в порядок»). Как видим,

Выясним, сколько всего можно составить размещений из элементов по без повторений. Составление размещения представим себе как последовательное заполнение мест, которые мы будем изображать в виде клеточек (рис. 126). На первое место мы можем выбрать один из п элементов заданного множества (то есть элемент для первой клеточки можно выбрать способами).

Если элементы нельзя повторять, то на второе место можно выбрать только один элемент из оставшихся, то есть из — 1 элементов. Теперь уже два элемента использованы и на третье место можно выбрать только один из — 2 элементов и т. д. На -e место можно выбрать только один из элементов.

Поскольку требуется выбрать элементы и на первое место, и на второе, . и на-e, то используем правило произведения, получим следующую формулу числа размещений из элементов по

Например, (что совпадает с соответствующим значением, полученным выше). Аналогично можно обосновать формулу для нахождения числа размещений с повторениями.

При решении простейших комбинаторных задач важно правильно выбрать формулу, по которой будут проводиться вычисления. Для этого достаточно выяснить следующее:

  • — Учитывается ли порядок следования элементов в соединении?
  • — Все ли заданные элементы входят в полученное соединение?

Если, например, порядок следования элементов учитывается и из заданных элементов в соединении используется только элементов, то по определению — это размещение из элементов по .

Заметим, что после определения вида соединения следует также выяснить, могут ли элементы в соединении повторяться, то есть выяснить, какую формулу необходимо использовать — для количества соединений без повторений или с повторениями.

Примеры решения задач:

Пример №42

На соревнования по легкой атлетике приехала команда из 12 спортсменок. Сколькими способами тренер может определить, кто из них побежит в эстафете 4 х 100 м на первом, втором, третьем и четвертом этапах?

Решение:

Количество способов выбрать из 12 спортсменок четырех для участия в эстафете равно количеству размещений из 12 элементов по 4 (без повторений), то есть

Для выбора формулы выясняем ответы на вопросы, приведенные выше. Поскольку для спортсменок важно, в каком порядке они будут бежать, то порядок при выборе элементов учитывается. В полученное соединение входят не все 12 заданных элементов. Следовательно, соответствующее соединение — размещение из 12 элементов по 4 (без повторений, поскольку каждая спортсменка может бежать только на одном этапе эстафеты).

Пример №43

Найдите количество трехзначных чисел, которые можно составить из цифр 1, 2, 3, 4, 5, 6, 7, если цифры в числе не повторяются.

Решение:

Количество трехзначных чисел, которые можно составить из семи цифр 1, 2, 3, 4, 5, 6, 7, равно числу размещений из 7 элементов по 3, то есть

Для выбора формулы выясняем, что для чисел, которые мы будем составлять, порядок следования цифр учитывается и не все элементы выбираются (только 3 из заданных семи). Следовательно, соответствующее соединение — размещение из 7 элементов по 3 (без повторений).

Пример №44

Найдите количество трехзначных чисел, которые можно составить из цифр 1, 2, 3, 4, 5, 6, 0, если цифры в числе не повторяются.

Выбор формулы проводится таким же образом, как и в задаче 2. Следует учесть, что если число, составленное из трех цифр, начинается цифрой О, то оно не считается трехзначным. Следовательно, для ответов на вопросы задачи можно сначала из заданных 7 цифр записать все числа, состоящие из 3 цифр (см. пример 2), а затем из количества полученных чисел вычесть количество чисел, составленных из трех цифр, но начинающих цифрой 0. В последнем случае мы фактически будем из всех цифр без нуля (их 6) составлять двузначные числа. Тогда их количество равно числу размещений из 6 элементов по 2 (см. решение).

Также можно выполнить непосредственное вычисление, последовательно заполняя три места в трехзначном числе и используя правило произведения. В этом случае удобно сделать рассуждения наглядными, изображая соответствующие разряды в трехзначном числе в виде клеточек, например, так:

  • 6 возможностей
  • 6 возможностей
  • 5 возможностей

Решение:

Количество трехзначных чисел, которые можно составить из семи цифр (среди которых нет цифры 0), если цифры в числе не повторяются, равно числу размещений из 7 элементов по 3, то есть

Но среди данных цифр есть цифра 0, с которой не может начинаться трехзначное число. Поэтому из размещений из 7 элементов по 3 необходимо исключить те размещения, в которых первым элементом является цифра 0. Их количество равно числу размещений из 6 элементов по 2, то есть Следовательно, искомое количество трехзначных чисел равно

Пример №45

Решите уравнение

Решение:

Тогда получаем На ОДЗ это уравнение равносильно уравнениям:

Уравнения, в запись которых входят выражения, обозначающие количество соответствующих соединений из х элементов, считаются определенными только при натуральных значениях переменной х. В данном случае, чтобы выражение имело смысл необходимо выбирать натуральные значения (в этом случае также существует и, конечно, Для преобразования уравнения используем соответствующие формулы:

Перестановки

Перестановкой из п элементов называется любое упорядоченное множество из элементов

Напомним, что упорядоченное множество — это такое множество, для которого указано, какой элемент находится на первом месте, какой на втором. какой на

Например, переставляя цифры в числе 236 (там множество цифр <2; 3; 6>уже упорядоченное), можно составить такие перестановки без повторений: (2; 3; 6), (2; 6; 3), (3; 2; 6), (3; 6; 2), (6; 2; 3), (6; 3; 2) — всего 6 перестановок*.

Количество перестановок без повторений из элементов обозначается (Р — первая буква французского слова permutation — перестановка). Как видим,

Фактически перестановки без повторений из элементов являются размещениями из элементов по без повторений, поэтому Произведение 1 • 2 • 3 •. • обозначается

!. Поэтому полученная формула числа перестановок без повторений из элементов может быть записана так:

*Отметим, что каждая такая перестановка определяет трехзначное число, составленное из цифр 2,3,6 так, что цифры в числе не повторяются.

Например, (что совпадает с соответствующим значением, полученным выше).

С помощью факториалов формулу для числа размещений без повторений

можно записать в другом виде. Для этого умножим и разделим выражение в формуле (1) на произведение Получаем

Следовательно, формула числа размещений без повторений из элементов по может быть записана так:

Для того чтобы этой формулой можно было пользоваться при всех значениях в частности, при договорились считать, что

Например, по формуле (2)

Обратим внимание, что в тех случаях, когда значение ! оказывается очень большим, ответы оставляют записанными с помощью факториалов.

Например,

Примеры решения задач:

Напомним, что для выбора формулы при решении простейших комбинаторных задач достаточно выяснить следующее:

  • — Учитывается ли порядок следования элементов в соединении?
  • — Все ли заданные элементы входят в полученное соединение? Если, например, порядок следования элементов учитывается и все п заданных элементов используются в соединении, то по определению это перестановки из п элементов.

Пример №46

Найдите, сколькими способами можно восемь учащихся построить в колонну по одному.

Решение:

Количество способов равно числу перестановок из 8 элементов. То есть

Для выбора соответствующей формулы выясняем ответы на вопросы, приведенные выше. Поскольку порядок следования элементов учитывается и все 8 заданных элементов выбираются, то соответствующие соединения — это перестановки из 8 элементов без повторений. Их количество можно вычислить по формуле.

Пример №47

Найдите количество разных четырехзначных чисел, которые можно составить из цифр 0, 3, 7, 9 (цифры в числе не повторяются).

Решение:

Из четырех цифр 0, 3, 7, 9, не повторяя заданные цифры, можно получить перестановок. Перестановки, начинающиеся с цифры 0, не являются записью четырехзначного числа — их количество . Тогда искомое количество четырехзначных чисел равно

Поскольку порядок следования элементов учитывается и для получения четырехзначного числа надо использовать все элементы, то искомые соединения — это перестановки из 4 элементов. Их количество — . При этом необходимо учесть, что в четырехзначном числе на первом месте не может стоять цифра 0. Таких чисел будет столько, сколько раз мы сможем выполнить перестановки из 3 оставшихся цифр, то есть .

Пример №48

Есть десять книг, из которых четыре — учебники. Сколькими способами можно поставить эти книги на полку так, чтобы все учебники стояли рядом?

Решение:

Сначала будем рассматривать учебники как одну книгу. Тогда на полке надо расставить не 10, а 7 книг. Это можно сделать способами. В каждом из полученных наборов книг можно выполнить еще перестановок учебников. По правилу умножения искомое количество способов равно

Задачу можно решать в два этапа. На первом этапе условно будем считать все учебники за 1 книгу. Тогда получим 7 книг (6 не учебников + 1 условная книга — учебник). Порядок следования элементов учитывается и используются все элементы (поставить на полку необходимо все книги). Следовательно, соответствующие соединения — это перестановки из 7 элементов. Их количество — .

На втором этапе решения будем переставлять между собой только учебники. Это можно сделать способами. Поскольку нам надо переставить и учебники, и другие книги, то используем правило произведения.

Сочетания без повторений

Сочетанием без повторений из элементов по называется любое -элементное подмножество -элементного множества.

Например, из множества > можно составить следующие сочетания без повторений из трех элементов:

Количество сочетаний без повторений из п элементов по к элементов обозначается символом (читается: «Число сочетаний из » или «це из », С — первая буква французского слова combinaison — сочетание). Как видим,

Выясним, сколько всего можно составить сочетаний без повторений из элементов по . Для этого используем известные нам формулы числа размещений и перестановок.

Составление размещения без повторений из элементов по проведем в два этапа. Сначала выберем разных элементов из заданного -элементного множества, не учитывая порядок выбора этих элементов (то есть выберем -элементное подмножество из -элементного множества — сочетание без повторений из -элементов по ). По нашему обозначению это можно сделать способами. После этого полученное множество из к разных элементов упорядочим. Его можно упорядочить способами. Получим размещения без повторений из элементов по . Следовательно, количество размещений без повторений из элементов по в раз больше числа сочетаний без повторений из элементов по . То есть Отсюда Учитывая, что по формуле (2) , получаем

Например, совпадает со значением, полученным выше.

Используя формулу (3), можно легко обосновать свойство 1 числа сочетаний без повторений, приведенное в таблице 21.

1) Поскольку

Для того чтобы формулу (4) можно было использовать и при , договорились считать, что. Тогда по формуле (4) .

Если в формуле (3) сократить числитель и знаменатель на, то получим формулу, по которой удобно вычислять при малых значениях :

Например,

Вычисление числа сочетаний без повторений с помощью треугольника Паскаля

Для вычисления числа сочетаний без повторений можно применять формулу (3): , а можно последовательно вычислять соответствующие значения, пользуясь таким свойством:

Для обоснования равенства (6) найдем сумму учитывая, что

, следовательно,

Это равенство позволяет последовательно вычислять значения с помощью специальной таблицы, которая называется треугольником Паскаля. Если считать, что , то таблица будет иметь следующий вид (табл. 23).

Каждая строка этой таблицы начинается с единицы и заканчивается единицей .

Если какая-либо строка уже заполнена, например, третья, то в четвертой строке надо записать на первом месте единицу. На втором месте запишем число, равное сумме двух чисел третьей строки, стоящих над ним левее и правее (поскольку по формуле (6).

На третьем месте запишем число, равное сумме двух следующих чисел третьей строки, стоящих над ним левее и правее, и т. д. (а на последнем месте снова запишем единицу).

Примеры решения задач:

Обратим внимание, что, как и раньше, для выбора формулы при решении простейших комбинаторных задач достаточно ответить на вопросы:

  1. Учитывается ли порядок следования элементов в соединении?
  2. Все ли заданные элементы входят в полученное соединение?

Для выяснения того, что заданное соединение является сочетанием, достаточно ответить только на первый вопрос. Если порядок следования элементов не учитывается, то по определению это сочетания из элементов по элементов.

Пример №49

Из 12 членов туристической группы надо выбрать трех дежурных. Сколькими способами можно сделать этот выбор?

Решение:

Количество способов выбрать из 12 туристов трех дежурных равно количеству сочетаний из 12 элементов по 3 (без повторений), то есть

Для выбора соответствующей формулы выясняем ответы на вопросы, приведенные выше. Поскольку порядок следования элементов не учитывается (для дежурных неважно, в каком порядке их выберут), то соответствующее соединение является сочетанием из 12 элементов по 3 (без повторений). Для вычисления можно использовать формулы (3) или (5), в данном случае применяем формулу (3):

Пример №50

Из вазы с фруктами, в которой лежит 10 разных яблок и 5 разных груш, требуется выбрать 2 яблока и 3 груши. Сколькими способами можно сделать такой выбор?

Решение:

Выбрать 2 яблока из 10 можно способами. При каждом выборе яблок груши можно выбрать способами. Тогда по правилу произведения выбор требуемых фруктов можно выполнить способами. Получаем

Сначала отдельно выберем 2 яблока из 10 и 3 груши из 5. Поскольку при выборе яблок или груш порядок следования элементов не учитывается, то соответствующие соединения — сочетания без повторений.

Учитывая, что требуется выбрать 2 яблока и 3 груши, используем правило произведения и перемножим полученные возможности выбора яблок() и груш ().

Бином Ньютона

Поскольку то формулу бинома Ньютона можно записать еще и так:

Общий член разложения степени бинома имеет вид

Коэффициенты называют биномиальными коэффициентами.

Свойства биномиальных коэффициентов:

  1. Число биномиальных коэффициентов (а следовательно, и число слагаемых в разложении степени бинома) равно
  2. Коэффициенты членов, равноудаленных от начала и конца разложения, равны между собой (поскольку
  3. Сумма всех биномиальных коэффициентов равна
  4. Сумма биномиальных коэффициентов, стоящих на четных местах, равна сумме биномиальных коэффициентов, стоящих на нечетных местах.
  5. Для вычисления биномиальных коэффициентов можно воспользоваться треугольником Паскаля, в котором вычисления коэффициентов основываются на формуле

В каждом ряду по краям стоят единицы, а каждое из остальных чисел равно сумме двух чисел, находящихся над ним справа и слева Например,

Объяснение и обоснование Бинома Ньютона

Двучлен вида а + х также называют биномом. Из курса алгебры известно, что:

Можно заметить, что коэффициенты разложения степени бинома при совпадают с числами в соответствующей строке треугольника Паскаля. Оказывается, что это свойство выполняется для любого натурального то есть справедлива формула:

Формулу (7) называют биномом Ньютона. Правая часть этого равенства называется разложением степени бинома называют биномиальными коэффициентами. Общий член разложения степени бинома имеет вид

Обосновать формулу (7) можно, например, следующим образом.

Если раскрыть скобки в выражении то есть умножить бином а + х сам на себя раз, то получим многочлен степени относительно переменной х. Тогда результат можно записать так:

Чтобы найти значение подставим в обе части равенства (8) вместо х значение 0. Получаем можем записать:

Чтобы найти сначала возьмем производную от обеих частей равенства (8):

затем, подставив в обе части полученного равенства (9) х = 0, получим: Учитывая, чтоможем записать: Аналогично, чтобы найти возьмем производную от обеих частей равенства (9):

и, подставив х = 0 в равенство (10), получим Тогда Другие коэффициенты находят аналогично. Если продифференцировать раз равенство (8), то получим:

Подставляя в последнее равенство х = 0, имеем

В каждом ряду по краям стоят единицы, а каждое из остальных чисел равно сумме двух чисел, находящихся над ним справа и слева

Умножим обе части равенства (11) на и найдем коэффициент

. Подставляя найденные значения

1, 2, . ) в равенство (8), получаем равенство (7).

Записывая степень двучлена по формуле бинома Ньютона для небольших значений п, биномиальные коэффициенты можно вычислять по треугольнику Паскаля (табл. 25, см. также табл. 24).

Например,

Так как формулу бинома Ньютона можно записать в виде:

а учитывая, что, еще и так:

Если в формуле бинома Ньютона (12) заменить х на (-х), то получим формулу возведения в степень разности а — х:

. Например, ( (знаки членов разложения чередуются!).

Свойства биномиальных коэффициентов

1. Число биномиальных коэффициентов (а следовательно, и число слагаемых) в разложении -й степени бинома равно + 1, поскольку разложение содержит все степени х от 0 до (и других слагаемых не содержит).

2. Коэффициенты членов, равноудаленных от начала и конца разложения, равны между собой, поскольку

3. Сумма всех биномиальных коэффициентов равна 2″.

Для обоснования полагаем в равенстве (13) (или в равенстве (7)) значения а = х = 1 и получаем

Например,

4. Сумма биномиальных коэффициентов, стоящих на четных местах, равна сумме биномиальных коэффициентов, стоящих на нечетных местах,

Для обоснования возьмем в равенстве (13) значения а =1, х = —1. Получаем

Тогда

Примеры решения задач:

Пример №51

По формуле бинома Ньютона найдите разложение степени

Для нахождения коэффициентов разложения можно использовать треугольник Паскаля или вычислять их по общей формуле. По треугольнику Паскаля коэффициенты равны: 1, 6, 15, 20, 15, б, 1. Учитывая, что при возведении в степень разности знаки членов разложения чередуются, получаем

Для упрощения записи ответа можно избавиться от иррациональности в знаменателях полученных выражений (см. решение) или сначала учесть, что ОДЗ заданного выражения: х > 0, и тогда То есть заданное выражение можно записать так: и возвести в степень последнее выражение.

Решение:

Пример №52

В разложении степени найти член, содержащий

Решение:

► ОДЗ: > 0. Тогда

Общий член разложения:

По условию член разложения должен содержать, следовательно,

. Отсюда

Тогда член разложения, содержащий , равен

На ОДЗ (b > 0) каждое слагаемое в заданном двучлене можно записать как степень с дробным показателем. Это позволит проще записать общий член разложения степени: (где = 0, 1, 2, . ), выяснить, какой из членов разложения содержит , и записать его.

Чтобы упростить запись общего члена разложения, удобно отметить, что

Зачем нужна комбинаторика

Для решения задач с использованием классического определения вероятности необходимо знать основные правила и формулы комбинаторики -раздела математики, изучающего методы решения комбинаторных задач — т.е. задач, связанных с подсчетом числа различных комбинаций.

Пусть — элементы конечного множества. Сформулируем два важных правила, часто применяемых при решении комбинаторных задач.

Правило суммы

Если элемент может быть выбран способами, элемент / способами, . элемент способами, то выбор одного из элементов может быть осуществлен пспособами.

Пример №53

В группе 30 студентов. Известно, что 5 из них на экзамене по математике получили оценку «отлично», 10 — оценку «хорошо», остальные -«удовлетворительно». Сколько существует способов выбрать одного студента, получившего на экзамене оценку «отлично» или «хорошо»?

Решение:

Студент, получивший оценку «отлично» может быть выбранспособами, оценку «хорошо» — способами. По правилу суммы существует способов выбора одного студента, получившего на экзамене оценку «отлично» или «хорошо».

Правило произведения

Если элемент может быть выбран способами, после этого элемент может быть выбран способами после каждого такого выбора элемент может быть выбран способами, то выбор всех элементов в указанном порядке может быть осуществлен способами.

Пример №54

В группе 30 студентов. Необходимо выбрать старосту, его заместителя и профорга. Сколько существует способов это сделать?

Решение:

Старостой может быть выбран любой из 30 студентов, его заместителем – любой из оставшихся 29, а профоргом – любой из оставшихся 28 студентов, т.е. По правилу произведения общее число способов выбора старосты, его заместителя и профорга равно = = 24360 способов. ◄

Пусть дано множество из n различных элементов. Из этого множества могут быть образованы подмножества из m элементов (0 ≤ m ≤n). Например, из 5 элементов a, b, c, d, e могут быть отобраны комбинации по 2 элемента – ab, bc, cd, ba и т.д., по 3 элемента – abc, cbd, cba и т.д.

Если комбинации из n элементов по m отличаются либо составом элементов, либо порядком их расположения (либо и тем и другим), то такие комбинации называют размещениями из n элементов по m. Число размещений из n элементов по m находится по формуле где n! равно произведению n первых чисел натурального ряда, т.е. n! = 1·2·…·n.

Пример №55

Сколько можно записать двузначных чисел, используя без повторения цифры от 1 до 5?

Решение:

В данном случае двузначное число является комбинацией из пяти цифр по две цифры. Поскольку числа отличаются как составом входящих в них цифр, так и порядком их расположения, то в данном случае двузначные числа являются размещениями из пяти цифр по две. Число таких размещений

Если комбинации из n элементов по m отличаются только с о с т а в о м элементов (порядок их расположения не имеет значения), то такие комбинации называют сочетаниями из n элементов по m.

Число сочетаний из n элементов по m находится по формуле

Пример №56

Необходимо выбрать в подарок две из пяти имеющихся различных книг. Сколькими способами можно это сделать?

Решение:

Из смысла задачи следует, что порядок выбора книг не имеет значения. Здесь важен только их состав. Поэтому в данном случае комбинации книг представляют собой сочетания из 5 книг по 2. Число таких комбинаций Если в размещениях из n элементов по m некоторые из элементов (или все) могут оказаться одинаковыми, то такие размещения называют размещениями с повторениями из n элементов по m. Число размещений с повторениями равно

Пример №57

Сколько можно записать трехзначных чисел, которые не содержат цифр 0 и 5?

Решение:

В данном случае трехзначное число является комбинацией из восьми цифр (0 и 5 не учитываются) по три цифры. При этом некоторые из цифр (или все) могут повторяться. Поэтому в данном случае трехзначные числа является размещениями с повторениями из восьми цифр по три. Число таких размещений с повторениями Если в сочетаниях из n элементов по m некоторые из элементов (или все) могут оказаться одинаковыми, то такие сочетания называют сочетаниями с повторениями из n элементов по m. Число сочетаний с повторениями равно где определяется по формуле (1.6).

Пример №58

В почтовом отделении продаются открытки восьми видов. Сколькими способами можно купить в нем три открытки?

Решение:

Учитывая, что порядок выбора открыток не имеет значения, а важен только их состав, причем некоторые из открыток (или все) могут оказаться одинаковыми, искомое число способов находим по формуле числа сочетаний с повторениями Если комбинации из n элементов отличаются только порядком расположения элементов, то такие комбинации называют перестановками из n элементов. Число перестановок из n элементов находится по формуле

Пример №59

Порядок выступления 5 участников конкурса определяется жребием. Сколько различных вариантов жеребьевки при этом возможно?

Решение:

Каждый вариант жеребьевки отличается только порядком участников конкурса, т.е. является перестановкой из 5 элементов. Их число равно Если в перестановках из общего числа n элементов есть k различных элементов, при этом 1-й элемент повторяется раз, 2-й элемент – раз, k-й элемент – раз, причем, то такие перестановки называют перестановками с повторениями из n элементов. Число перестановок с повторениями равно

Пример №60

Сколько можно составить шестизначных чисел, состоящих из цифр 3, 5, 7, в которых цифра 3 повторяется 3 раза, цифра 5 – 2 раза, цифра 7 – 1 раз?

Решение:

Каждое шестизначное число отличается от другого порядком следования цифр (причем а их сумма равна 6), т.е. является перестановкой с повторениями из 6 элементов. Их число равно

Рекомендую подробно изучить предметы:
  1. Теория вероятностей
  2. Математическая статистика
Ещё лекции с примерами решения и объяснением:
  • Классическое определение вероятности
  • Геометрические вероятности
  • Теоремы сложения и умножения вероятностей
  • Формула полной вероятности
  • Математическая обработка динамических рядов
  • Корреляция — определение и вычисление
  • Элементы теории ошибок
  • Методы математической статистики

При копировании любых материалов с сайта evkova.org обязательна активная ссылка на сайт www.evkova.org

Сайт создан коллективом преподавателей на некоммерческой основе для дополнительного образования молодежи

Сайт пишется, поддерживается и управляется коллективом преподавателей

Whatsapp и логотип whatsapp являются товарными знаками корпорации WhatsApp LLC.

Cайт носит информационный характер и ни при каких условиях не является публичной офертой, которая определяется положениями статьи 437 Гражданского кодекса РФ. Анна Евкова не оказывает никаких услуг.


источники:

http://urokimatematiki.ru/konspekt-uroka-na-temu-reshenie-kombinatornih-uravneniy-klass-5297.html

http://www.evkova.org/kombinatorika